Chapter 7: Lipid and Ethanol Metabolism

Réussis tes devoirs et examens dès maintenant avec Quizwiz!

*The answer is C.* A lack of apolipoprotein CII would mean that lipoprotein lipase could not be activated, and the triglyceride in both chylomicrons and VLDL would be unable to be digested. This would lead to elevated levels of these particles, and a very high serum triglyceride level. Since VLDL is not being converted to IDL or LDL cholesterol levels are not elevated. Defects in either apo B100 or apo B48 would lead to a loss of either VLDL or chylomicrons, which is not observed. A defect in pancreatic lipase would lead to steatorrhea, as the dietary triglycerides would not be able to be digested. A defect in LCAT would affect HDL metabolism, but not triglyceride metabolism.

A 12-year-old female presented with severe abdominal pain and was found to have a markedly elevated plasma triglyceride concentration (750 mg/dL). A lipoprotein analysis revealed elevated levels of chylomicrons and VLDL and reduced levels of HDL. Which protein might be defective in this patient? (A) Apo B100 (B) Apo B48 (C) Apo CII (D) Pancreatic lipase (E) LCAT

*The answer is C.* The two major groups of lysosomal storage disease are sphingolipidoses and mucopolysaccharidoses. An absence of α-L-iduronidase, as in Hurler's syndrome and Scheie's syndrome, leads to accumulations of dermatan sulfate and heparan sulfate. Scheie's syndrome is less severe, with corneal clouding, joint degeneration, and increased heart disease. Hurler's syndrome has the same symptoms plus mental and physical retardation leading to early death. The later onset in this child is compatible with a diagnosis of Scheie's syndrome. Note that Hurler's and Scheie's syndromes result from mutations at the same locus—hence their identical McKusick numbers. The reasons for the differences in disease severity are unknown. All of the other enzyme deficiencies listed lead to the lack of proper breakdown of sphingolipids and their accumulation as gangliosides, glucocerebrosides, and sphingomyelins. Symptoms of lipidoses may include organ enlargement, mental retardation, and early death.

A 15-year-old boy has a long history of school problems and is labeled as hyperactive. His tissues are puffy, giving his face a "coarse" appearance. His IQ tests have declined recently and are now markedly below normal. Laboratory studies demonstrate normal amounts of sphingolipids in fibroblast cultures with increased amounts of glycosaminoglycans in urine. Which of the following enzyme deficiencies might explain the boy's phenotype? a. Hexosaminidase A b. Glucocerebrosidase c. α-L-iduronidase d. α-galactocerebrosidase e. β-gangliosidase A

*The answer is D.* The patient has sitosterolemia, an accumulation of plant sterols (phytosterols) in cells and tissues. Under normal conditions, phytosterols can diffuse into the epithelial cells, but they are actively transported back into the intestinal lumen by an ABC-cassette (ATP-binding) containing protein, ABCG5 (the other protein responsible for phytosterol efflux is ABCG8).Those sterols which make it to the liver are exported by the same proteins in the liver to the bile duct, where they will be released along with the bile during fat digestion. In the absence of activity of either ABCG5 or ABCG8, the phytosterols are packaged into chylomicrons and are eventually delivered to the liver, where they are packaged into VLDL. While human cells cannot utilize phytosterols, their increased presence interferes with the synthesis of cholesterol and the normal cholesterol recycling within the affected patient. Patients with this disorder develop premature coronary artery disease. It has been hypothesized that the high levels of plant sterols in the circulating lipoprotein particles accelerate the deposition of these sterols in the walls of the arteries, promoting atherosclerosis. This disorder is not due to mutations in either apo B100 or apo B48, as both VLDL and chylomicrons are synthesized normally in the patient. The defect is not in ABC1, as the patient does not display the symptoms of Tangier disease. The defect is also not in MTTP, as a defect in that protein leads to abetalipoproteinemia.

A 16-year-old male presents to you with xanthomas on the extensor tendons of the hand and Achilles tendon and arthritis of the knees. He has had one previous heart attack, despite normal cholesterol levels. Further analysis of his serum showed greatly elevated levels of plant sterols. The molecular defect in this patient is most likely in which of the following proteins? (A) Apo B100 (B) Apo B48 (C) ABC1 (D) ABCG5 (E) MTTP

*The answer is D.* Upon entry into the playground to have fun, the child secretes epinephrine, which results in adipose tissue triglyceride (TG) breakdown and entry of fatty acids (FA) into muscle and liver for mitochrondrial β-oxidation. A defect in the carnitine shuttle system in this patient would result in accumulation of TG (re-synthesis from FA) in liver and muscle cytoplasm.

A 3-year-old child complains of muscle pain and weakness while in the playground and is admitted to the hospital for examination. Tests reveal slight hepatomegaly and cardiomegaly. A liver biopsy shows extreme but nonspecific fatty changes, and a muscle biopsy contains large amounts of cytoplasmic vacuoles containing neutral lipid. A one-day fast is performed and shows a drop in blood glucose levels without a corresponding production of ketone bodies. The pH of the blood is nor-mal. Which of the following diagnoses might account for this child's problems? A. Bilirubin diglucuronide transporter deficiency B. Glucose 6-phosphatase deficiency C. Mitochondrial 3-hydroxy 3-methylglutaryl-CoA synthase deficiency D. Systemic carnitine deficiency E. Vitamin D deficiency

*The answer is C.* Apo A-1 is present on the surface of HDL and functions to activate circulating LCAT (lecithin:cholesterol acyltransferase) to esterify cholesterol, using lecithin (phosphatidylcholine) as the fatty acid donor. Esterification of cholesterol in blood occurs to trap the resultant cholesterol esters in HDL where they can subsequently be transferred to other lipoproteins (IDL) or taken up directly by hepatocytes and steroidogenic tissues.

A 20-year-old man is taken to the university clinic to determine the cause of recurring hyperlipidemia, proteinuria, and anemia. Fasting blood tests reveal slightly elevated concentrations of unesterified cholesterol and phosphatidylcholine. The patient is given a 100 gram chocolate bar and blood lipid levels are monitored hourly. Results reveal significantly increased levels of unesterified cholesterol and phosphatidylcholine for extended periods. Deficiency of which of the following proteins is most likely to be associated with these observations? A. Acyl-CoA:cholesterol B. acyltransferase (ACAT) C. Apoprotein A-1 D. Apoprotein B48 E. Apoprotein B100 F. Lipoprotein lipase

*The answer is C.* The adult form of Gaucher disease causes hepatosplenomegaly, osteoporosis of the long bones, and the characteristic wrinkled appearance of the cytosol of cells. This is also the sphingolipidosis in which glucosylceramides accumulate. The deficient enzyme is β-glucosidase (a glucocerebrosidase).

A 25-year-old woman with a history that included hepatosplenomegaly with eventual removal of the spleen, bone and joint pain with several fractures of the femur, and a liver biopsy that showed wrinkled-looking cells with accumulations of glucosylceramides was presented at Grand Rounds. The likely diagnosis for this patient is: A. Fabry disease. B. Farber disease. C. Gaucher disease. D. Krabbe disease. E. Niemann-Pick disease.

*The answer is B.* The findings are indicative of heterozygous type lla familial hypercholesterolemia, an autosomal dominant disease. Deficient CETP, LCAT or fatty acid-CoA synthetase would not elevate LDL cholesterol. VLDL are not produced from LDL.

A 42-year-old man presents with a chief complaint of intermittent claudication during exercise. His family history is significant for the presence of cardiovascular disease on his father's side, but not on his mother's side. Physical exam reveals xanthelasmas and bilateral tendon xanthomas. A plasma lipid profile reveals cholesterol level 340 mg/dL, with a high LDL/HDL ratio. He is given instructions for dietary modifications and a prescription for simvastatin. The clinical findings noted in this patient are most likely caused by deficient production of A. lethicin cholesterol B. acyltransferase C. apoB-100 receptors D. fatty acyl-CoA synthetase E. VLDL from LDL F. cholesterol ester transfer protein

*The answer is B.* Untreated ethylene glycol of antifreeze can be converted to the kidney toxin oxalate crystals. This occurs by oxidation of ethylene glycol. The first committed step in this process is the oxidation of ethylene glycol to an aldehyde by alcohol dehydrogenase. This is normally the route for converting ethanol (drinking alcohol) to acetate. Patients who have ingested ethylene glycol or wood alcohol (methanol) are placed on a nearly intoxicating dose of ethanol by a nasogastric tube together with intravenous saline and sodium bicarbonate. This treatment is carried out intermittently along with hemodialysis until no traces of ethylene glycol are seen in the blood. Ethanol acts as a competitive inhibitor of alcohol dehydrogenase with respect to ethylene glycol or methanol metabolism.

A 3-year-old child is brought into the ER while you are on duty. She is cold and clammy and is breathing rapidly. She is obviously confused and lethargic. Her mother indicates she has accidentally ingested automobile antifreeze while playing in the garage. Following gastrointestinal lavage and activated charcoal administration, one of the treatments you immediately initiate involves a. Intravenous infusion of oxalic acid b. Nasogastric tube for ethanol administration c. Flushing out the bladder via a catheter d. Intramuscular injection of epinephrine e. Simply waiting and measuring vital signs

*The answer is B.* Ethanol inhibits the drug detoxifying system for barbiturates; thus, in the presence of ethanol, the high levels of barbiturates being taken (due to the tolerance) are now toxic (the system that breaks down the drug has been inhibited). Ethanol does not increase absorption of the drug from the digestive tract, nor does acetaldehyde, ethanol's oxidation product, react with barbiturates. Barbiturate action is not affected by energy production (acetyl-CoA). The ethanol inhibition of cytochrome P450 systems is also not due to ethanol's dehydration effect.

A 34-year-old man was prescribed barbiturates 6 months ago for a seizure disorder. However, with time, the physician has had to increase his daily dosage to maintain the same therapeutic drug level. One night, the patient consumes a large amount of alcohol. He continues to take his usual dose of seizure medication. He dies that night in his sleep. This is due to which of the following? (A) Ethanol stimulating barbiturate absorption by the stomach (B) Ethanol inhibition of a cytochrome P450 system (C) Acetaldehyde reacting with the drug, creating a toxic compound (D) Acetyl-CoA production leads to enhanced energy production, which synergizes with barbiturate action (E) Ethanol's dehydration effect leads to toxic concentrations of the seizure medication in the blood

*The answer is D.* Barbiturates are xenobiotics, and the body induces specific cytochrome P450 systems to help detoxify and excrete the barbiturates. When the man first begins taking the drug, a low concentration of drug is sufficient to exert a physiological effect, as the drug detoxifying system has not yet been induced. As the detoxifying system is induced, however, higher concentrations of drug are required to have the same effect, as the rate of excretion of the drug is increased as the detoxification system is induced. The "tolerance" to drugs, in this case, is not due to downregulation of drug receptors or decreased absorption of the drug from the stomach. There is no induction of target gene expression, leading to enhanced drug action, nor are opposing neurotransmitters expressed. The tolerance comes about due to enhanced inactivation of the drug due to the induction of drug-metabolizing enzymes.

A 34-year-old man was prescribed barbiturates 6 months ago for a seizure disorder. However, with time, the physician has had to increase his daily dosage to maintain the same therapeutic drug level. This is due to which of the following? (A) Downregulation of drug receptors on the cell surface (B) Decreased absorption of the drug from the stomach (C) Increased synthesis of opposing neurotransmitters (D) Induction of drug-metabolizing enzymes (E) Induction of targeted enzyme synthesis

*The answer is D.* The man had eaten the unripe fruit of the ackee tree (from Jamaica). The unripened fruit contains the toxin hypoglycin A, which will interfere with carnitine's ability to transport acyl-carnitine groups across the inner mitochondrial membrane. This leads to a complete shutdown of fatty acid oxidation in all tissues in the affected individual, leading to severe hypoglycemia. Hypoglycin has no effect on fatty acid release from the adipocyte, or fatty acid entry into liver cells. Fatty acid oxidation is not directly inhibited, nor does this toxin directly inhibit the complexes of the electron transport chain and the proton-translocating ATPase.

A 35-year-old man in New York city, originally from Jamaica, purchased an illegally imported fruit from a street vendor and, within 4 h of eating the fruit, began vomiting severely. When brought to the emergency department the man was severely dehydrated and exhibited several seizures. The toxic effects of the fruit were interfering with which of the following? (A) Fatty acid release from the adipocyte (B) Fatty acid entry into the liver cell (C) Fatty acid activation (D) Fatty acid transport into the mitochondria (E) Oxidative phosphorylation

*The answer is A.* The child has Krabbe disease, a mutation in a galactosidase, which cannot remove galactose from galactose cerebroside (an inability to break the bond between galactose and ceramide). The buildup of galactose-ceramide leads to the neuronal and muscle damage seen in the child. An inability to degrade a sulfatide would lead to metachromatic leukodystropy, which has very different symptoms than Krabbe disease. An inability to degrade glucosylceramide leads to Gaucher disease, again, with a very different disease progression than that seen with Krabbe disease. A defect in the degradation of sphingomyelin leads to Niemann-Pick disease, with a different set of symptoms than that seen with Krabbe disease. A defect in degrading ceramide leads to Farber disease, a defect in ceramidase. Farber disease is similar to Krabbe disease, but often presents with hepatomegaly and splenomegaly.

A 4-month-old infant is brought to the pediatrician for a variety of problems. The child is frequently irritable, small for age, vomits frequently, and displays hypotonia, as well as hyperesthesia (auditory, tactile, and visual). Liver and spleen size are normal. As the child ages, his condition worsens, with rapid psychomotor deterioration, seizures, and blindness. This disorder is caused by an accumulation of which of the following in neuronal lysosomes? (A) Galactosylceramide (B) Sulfatide (C) Glucosylceramide (D) Sphingomyelin (E) Ceramide

*The answer is D.* Secretin is released from the intestine when food enters, and it signals the pancreas to release a watery mixture of bicarbonate into the intestine, in order to help neutralize the acid present from the digestion that occurred in the stomach. If the pH of the intestinal lumen is too low, the bile salts will not be ionized, and emulsification of the dietary fats will be inefficient, as will be the formation of mixed micelles to allow intestinal absorption of fat components. Digestion of carbohydrates and protein is not dependent on bile salt ionization. A loss of cholecystokinin would result in no pancreatic zymogens being secreted, and there would be no digestion of carbohydrates, proteins, or lipids within the intestine. A lack of insulin secretion, or glucagon secretion, does not affect digestion in the intestinal lumen. Cortisol secretion also does not alter intestinal digestion of nutrients.

A 43-year-old woman presents with steatorrhea. Fecal analysis reveals the presence of elevated triglycerides, phospholipids, and cholesterol esters. Levels of carbohydrate and protein were normal. Physical exam is unremarkable. A possible defect in the release of which of the following would lead to these results? (A) Cholecystokinin (B) Insulin (C) Glucagon (D) Secretin (E) Cortisol

*The answer is B.* Coenzyme Q is derived from isoprene units, which are produced in the pathway of cholesterol biosynthesis, after the HMG-CoA reductase step. If HMG-CoA reductase is inhibited (as it is by statins), then the production of the isoprenes is also reduced, and both Coenzyme Q and dolichol levels could become limiting. The biosynthesis of heme, ketone bodies, glycogen, or dihydrobiopterin is not dependent on isoprene units.

A 44-year-old man displayed elevated cholesterol levels and was prescribed a statin to reduce such levels. Statin treatment has the potential to interfere with the synthesis of which of the following? (A) Heme (B) Coenzyme Q (C) Ketone bodies (D) Glycogen (E) Dihydrobiopterin

*The answer is B.* Mevastatin, an analogue of mevalonic acid, acts as a feedback inhibitor of 3′-hydroxy-3′-methylglutaryl CoA (HMG-CoA) reductase, the regulated enzyme of cholesterol synthesis. Effective treatment with mevastatin, along with a low-fat diet, decreases levels of blood cholesterol. The lowering of cholesterol also lowers the amounts of the lipoprotein that transports cholesterol to the peripheral tissues, lowdensity lipoprotein (LDL). Since lipids like cholesterol and triglycerides are insoluble in water, they must be associated with lipoproteins for transport and salvage between their major site of synthesis (liver) and the peripheral tissues. Those lipoproteins associated with more insoluble lipids thus have lower density during centrifugation , a technique that separates the lowest-density chylomicrons from very-low-density lipoproteins (VLDLs with pre-β lipoproteins), low-density lipoproteins (LDLs with β-lipoproteins), intermediate-density lipoproteins (IDLs), and high-density lipoproteins (HDLs with α-lipoproteins). Each type of lipoprotein has typical apolipoproteins such as the apo B100 and apo B48 (translated from the same messenger RNA) in LDL. LDL is involved in transporting cholesterol from the liver to peripheral tissues, while HDL is a scavenger of cholesterol. The ratio of HDL to LDL is thus a predictor of cholesterol deposition in blood vessels, the cause of myocardial infarctions (heart attacks). The higher the HDL/LDL ratio, the lower the rate of heartattacks.

A 45-year-old man has a mild heart attack and is placed on diet and mevastatin therapy. Which of the following will be a result of this therapy? a. Low blood glucose b. Low blood LDLs c. High blood cholesterol d. High blood glucose e. Low oxidation of fatty acids f. Ketosis g. Lipolysis

*The answer is B.* Oxidized LDL is taken up by macrophages, which eventually turn into foam cells in the development of an atherosclerotic plaque. The higher one's LDL levels are, the more likely that oxidized LDL will form, leading to plaque formation. The receptor which recognizes and takes up oxidized LDL, SR-A1, is not downregulated, so the macrophage has an unlimited capacity to take up and store the oxidized LDL. Plaque formation does not occur due toelevated levels of nonoxidized LDL, HDL of any form, or triglycerides. A cartoon depiction of a normal and an atherosclerotic artery is shown below.

A 46-year-old man has been progressively having trouble breathing while walking. Walking from his car to his office has become difficult, and he has to stop to rest along the way. He visits his physician, who orders an angiogram, which shows blockage of major arteries leading to the heart. An initiating factor for the development of the blockage is which of the following? (A) LDL (B) Oxidized LDL (C) Triglycerides (D) HDL (E) Oxidized HDL

*The answer is B.* The man is suffering from acetaminophen poisoning. MEOS (the microsomal ethanol oxidizing system, also named CYP2E1) will convert acetaminophen into a toxic intermediate. In a chronic alcoholic, the MEOS has been induced and is very active. The toxic intermediate (NAPQI) can be rendered inactive by adding glutathione to the compound for safe excretion, and glutathione is a mercaptan (a compound with a free sulfhydryl group). Individuals with acetaminophen poisoning are given N-acetyl cysteine as a mechanism to increase glutathione production. Iron and vitamin C will not aid in detoxifying the toxic intermediate. Rifampin blocks RNA polymerase in bacteria. Aspirin will block cyclooxygenase, but will not stimulate NAPQI excretion.

A 52-year-old man has had bouts of alcohol abuse in his past. During his binges, he takes acetaminophen to help control some muscle pain. He then gets very ill (nausea, vomiting, and right upper quadrant pain), and is rushed to the emergency department. A potential treatment for this patient's symptoms is to take which of the following? (A) Aspirin (B) A mercaptan (C) Rifampin (D) Iron (E) Vitamin C

*The answer is B.* Ezetimibe reduces circulating cholesterol levels by blocking cholesterol absorption in the intestine, which is similar to the mechanism of action of phytosterols. Atorvastatin is a statin, and its mechanism of action is inhibition of HMG-CoA reductase. Pravastatin is also a statin and works as does atorvastatin. Simvastatin is yet another statin. Metformin is a lipid- and glucose-lowering drug which works via activation of the AMP-activated protein kinase and does not alter cholesterol absorption in the intestine.

A 57-year-old man has been taking low-dose aspirin to reduce his risk of heart disease. He adds phytosterols to his daily regime. Phytosterols have the same general mechanism of action as which of the following drugs? (A) Atorvastatin (B) Ezetimibe (C) Pravastatin (D) Simvastatin (E) Metformin

*The answer is B.* . Phytosterols interfere with cholesterol absorption in the intestine (through blockage of cholesterol incorporation into the mixed micelles, which are necessary for intestinal epithelial cells to absorb dietary cholesterol), thereby leading to a reduction in circulating cholesterol levels. The phytosterols do not interfere with the biosynthesis of cholesterol, nor do they alter the secretion of insulin. Phytosterols are also not capable of altering the rate of fatty acid biosynthesis, nor do they affect circulating triglyceride levels. The effect of phytosterols is specifi c for the inhibition of cholesterol absorption from the intestine.

A 57-year-old man has been taking low-dose aspirin to reduce his risk of heart disease. He adds phytosterols to his daily regime for which of the following? (A) To reduce circulating triglyceride levels (B) To reduce circulating cholesterol levels (C) To reduce endogenous cholesterol synthesis (D) To decrease insulin secretion (E) To reduce fatty acid biosynthesis

*The answer is C.* The child has a form of Gaucher disease, which is a defect in a glucosidase which removes glucose from glucosylceramide. The accumulation of glucosylcerebroside in the lysosomes leads to the observed symptoms. Defects in degrading galactosylceramide lead to Krabbe disease, which does not result in hepatomegaly and splenomegaly. A defect in degrading sulfatide leads to metachromatic leukodystrophy, which has different symptoms than what the child is experiencing. A defect in the degradation of sphingomyelin leads to Niemann-Pick disease, with a different set of symptoms than that seen with Gaucher disease. A defect in degrading ceramide leads to Farber disease, a defect in ceramidase, with more severe symptoms than those observed in Gaucher disease.

A 6-month-old boy is brought to the pediatrician due to a large stomach. The doctor noticed splenomegaly, with no pain. The boy was always tired and had anemia. The boy also has thrombocytopenia and bruises easily. X-rays show a deformity of the distal femur, as shown below. This disorder is caused by an accumulation of which of the following in macrophage lysosomes? (A) Galactosylceramide (B) Sulfatide (C) Glucosylceramide (D) Sphingomyelin (E) Ceramide

*The answer is A.* Impaired oxidation of fatty acids less than 12 carbons in length results in decreased production of acetyl CoA, the allosteric activator of pyruvate carboxylase, a gluconeogenic enzyme; thus, glucose levels fall. Acetyl CoA cannot be used for the net synthesis of glucose. Acetoacetate is a ketone body, and with MCAD deficiency ketogenesis is decreased. Impaired fatty acid oxidation means that less ATP and NADH are made, and both are needed for gluconeogenesis.

A 6-month-old boy was hospitalized following a seizure. History revealed that for several days prior, his appetite was decreased due to a "stomach virus". At admission, his blood glucose was 24 mg/dl (age-referenced normal is 60-100). His urine was negative for ketone bodies, but positive for a variety of dicarboxylic acids. A tentative diagnosis of medium-chain fatty acyl CoA dehydrogenase (MCAD) deficiency is made. In patients with MCAD deficiency, the fasting hypoglycemia is a consequence of: A. decreased acetyl CoA production. B. decreased ability to convert acetyl CoA to glucose. C. increased conversion of acetyl CoA to acetoacetate. D. increased production of ATP and NADH.

*The answer is D.* Malonyl CoA (three carbons) is synthesized from acetyl CoA (two carbons) by the addition of CO2, using the enzyme acetyl CoA carboxylase. Because CO2 is subsequently removed during fatty acid synthesis, the radioactive label will not appear at any position in newly synthesized fatty acids.

A low level of carbon dioxide labeled with 14C is accidentally released into the atmosphere surrounding industrial workers as they resume work following the lunch hour. Unknowingly, they breathe the contaminated air for 1 hour. Which of the following compounds will be radioactively labeled? A. All of the carbon atoms of newly synthesized fatty acid. B. About one half of the carbon atoms of newly synthesized fatty acids. C. The carboxyl atom of newly synthesized fatty acids. D. About one third of the carbons of newly synthesized malonyl CoA. E. One half of the carbon atoms of newly synthesized acetyl CoA.

*The answer is A.* The cherry-red spot is indicative of either Tay-Sachs disease (an autosomal recessive disorder leading to a loss of hexosaminidase A [hex A] activity) or Sandhoff disease (an autosomal recessive disorder leading to a loss of both hexosaminidase A and B [hex B] activity). With just a loss of hex A activity, GM2 would accumulate. With a loss of hex B activity, globoside and GM2 would accumulate. Thus, by measuring the levels of GM2 and globoside, one can distinguish between Tay-Sachs and Sandhoff disease. A loss of either hex A or hex B would not affect GM1 or GM3 degradation.

A 9-month-old child is taken to the pediatrician for lethargy and poor feeding. The physician notes a cherry-red spot in the child's retina. The baby seemed fine for the first three to six months, then began to have problems swallowing, overreacted to loud sounds, seemed to have problems with her vision, and began losing muscle mass and strength. Measurements of which two metabolites is critical to correctly diagnose this disorder? (A) GM2 and globoside (B) GM2 and GM3 (C) GM1 and globoside (D) GM1 and GM2 (E) Globoside and sphingomyelin

*The answer is D.* The hexosaminidase A gene encodes the hex A protein, and the hexosaminidase B gene encodes the hex B protein. Hexosaminidase A activity requires a complex of hex A and hex B proteins; hexosaminidase B activity only requires a complex of hex B proteins. Tay-Sachs disease is a defect in the hex A protein, affecting only hex A activity. Sandhoff disease is a defect in the hex B protein, which affects both hex A and hex B activity, due to the sharing of a common subunit between the two proteins. hex A and hex B are not in an operon (which is only operative in bacteria); in fact, the genes are on different chromosomes. There is an activating protein for hex A activity, but not hex B activity (a loss of the activating protein is known as Sandhoff activator disease, with symptoms very similar to Tay- Sachs disease). There are no mutations in transcriptional control proteins (either an activator or inhibitor) in Tay-Sachs or Sandhoff disease.

A 9-month-old child is taken to the pediatrician for lethargy and poor feeding. The physician notes a cherry-red spot in the child's retina. The baby seemed fine for the first three to six months, then began to have problems swallowing, overreacted to loud sounds, seemed to have problems with her vision, and began losing muscle mass and strength. Multiple enzymatic activities are lost. This is due to which of the following? (A) A common operon for the two genes contains a mutation in the promoter region (B) An inactivating mutation in an activator for the lost enzymatic activities (C) A transcriptional activator is inactivated (D) A common mutated subunit is present in the multiple activities (E) A transcriptional repressor is activated

*The answer is E.* Tay-Sachs is a defect in hexosaminidase A, which removes the terminal N-acetylgalactosamine residue from ganglioside GM2, producing the free sugar and GM3. Hexosaminidase A does not cleave glucose, ceramide, sphingosine, or the fatty acyl component of ceramide from GM2; it is specific for N-acetylgalactosamine.

A child has been diagnosed with Tay-Sachs disease, in which a particular lipid accumulates within the lysosomes. The component of this lipid which cannot be removed in the lysosome is which of the following? (A) Ceramide (B) Sphingosine (C) Fatty acid (D) Glucose (E) N-acetylgalactosamine

*The answer is D.* Fasted MCAD patients typically present with nonketotic hypoglycemia, lactic acidosis, and plasma dicarboxylates.

A child is diagnosed with a congenital deficiency of medium-chain acyl-CoA dehydrogenase activity. Which of the following signs or symptoms would most likely occur upon fasting in this child? A. Hypolacticacidemia B. Ketoacidosis C. Hyperglycemia D. Dicarboxylic acidosis E. Hyperchylomicronemia

*The answer is C.* The patient is experiencing the symptoms of vitamin B1 (thiamine) deficiency. Ethanol blocks thiamine absorption from the gut, so in the United States, one will usually only see a B1 deficiency in chronic alcoholics. One assay for B1 deficiency is to measure transketolase activity (which requires B1 as an essential cofactor) in the presence and absence of added B1. If the activity level increases when B1 is added, a vitamin deficiency is assumed. None of the other enzymes listed (transaldolase, aldolase, β-ketothiolase, and acetylcholine synthase) require B1 as a cofactor, and, thus, could not be used as a measure of B1 levels. A reaction catalyzed by transketolase is shown below (note the breakage of a carbon-carbon bond, and then the synthesis of a carbon-carbon bond to generate the product of the reaction).

A chronic alcoholic presents to the emergency department with nystagmus, peripheral edema, pulmonary edema, ataxia, and mental confusion. The physician orders a test to determine if there is a vitamin deficiency. An enzyme used for such a test can be which of the following? (A) Transaldolase (B) Aldolase (C) Transketolase (D) β-ketothiolase (E) Acetylcholine synthase

*The answer is A.* Patients with type I lipoprotein lipase deficiency are not able to rapidly delipidate chylomicrons, which carry dietary triacylglycerides, or VLDLs, which carry lipids packaged by the liver. The electrophoretic pattern (left lane in the figure) after fasting is thus similar to that of a normal patient after a meal—the chylomicrons (band A), LDLs with β lipoproteins (band B), VLDLs with pre-β- lipoproteins (band C), and HDLs with α-lipoproteins (band D) are all present because chylomicrons and VLDLs are not degraded normally. Hepatic lipase, ordinarily released by the liver to deal with chylomicron remnant and HDL metabolism, must slowly deal with the lipemia (lipoprotein buildup in blood) caused by the lack of lipoprotein lipase. In addition to the lipemia, steatorrhea (fatty stools) and stomach cramps are symptoms of lipoprotein lipase deficiency. A well-controlled low-fat diet is part of the therapy. In contrast, the electrophoretic pattern of patients with type IIa familial hypercholesterolemia (right lane) can appear, at first glance, to be normal, since only the LDL and HDL bands are expected following an overnight fast. However, closer inspection reveals an abnormally high accumulation of LDLs (band G), which cause the hypercholesterolemia. Since LDLs are a final stage in the catabolism of VLDLs, the high load of cholesterol being transported from the liver may cause some VLDLs to be present after fasting (band H) along with HDL scavenger lipoproteins (band I). In patients heterozygous for the disease, approximately one-half the normal amount of B100 LDL receptors are present. In homozygous patients, no B100 LDL receptors are present and most (∼70%) of the LDL must be cleared by the liver. In such patients, blood cholesterol levels are extraordinarily high and lead to profound early atherosclerosis and death.

A control and two patients with hyperlipidemia are studied after an overnight fast. Their plasma lipoprotein electrophoresis patterns are shown below, the control being in the middle lane. One of the patients has a pattern typical of type I lipoprotein lipase deficiency, and the other of type IIa familial hypercholesterolemia. Which of the bands observed in the electrophoretic gel patterns represents a lipoprotein fraction that is abnormally abundant after fasting and that is most enriched in triacylglycerides? a. Band A b. Band B c. Band C d. Band D e. Band E f. Band F g. Band G h. Band H i. Band I

*The answer is E.* Lithium primarily inhibits the phosphatase which converts inositol phosphate to free inositol, thereby disrupting the phosphatidylinositol cycle, leading to increased levels of the intermediates of the cycle, which are often signaling molecules. Lithium does not affect the generation of diacylglycerol, inositol trisphosphate (IP3), inositol bisphosphate (IP2), or inositol phosphate (IP); it affects just the conversion of IP to free inositol and a phosphate.

A depressed patient is prescribed lithium by his psychiatrist. The effect of lithium is to block the generation of which of the following? (A) Diacylglycerol (B) Inositol trisphosphate (C) Inositol bisphosphate (D) Inositol phosphate (E) Inositol

*The answer is C.* In Sandhoff disease, both hex A and hex B activities are lost. The mutation in Sandhoff disease does not affect sphingomyelinase activity, and there is no hexosaminidase C activity. Sandhoff disease is one of many which affect sphingolipid metabolism.

A diagnosis of Sandhoff disease was made. This results in a loss of which of the following enzymatic activities? (A) Hexosaminidase A and Hexosaminidase C (B) Hexosaminidase B and Hexosaminidase C (C) Hexosaminidase A and Hexosaminidase B (D) Hexosaminidase A and sphingomyelinase (E) Hexosaminidase B and sphingomyelinase

*The answer is A.* The primary reason for synthesizing conjugated bile acids is to lower the pKa of the acid, so that a higher percentage of the acid will be ionized in the intestine. The greater a bile acid is ionized, the more efficient the emulsification is for the digestion of the triglyceride. Without conjugation with glycine or taurine, the pKa of the bile salts is about 6.0; at a pH of 6.0, only 50% of the bile salts will be ionized in the intestinal lumen, which would produce inefficient triglyceride digestion, and the triglyceride content of the stool would increase. By reducing the pKa to 4.0 (conjugated with glycine) or 2.0 (conjugated with taurine), greater than 99% of the bile acids will be ionized, and triglyceride digestion will be maximal. If an inability to conjugate the bile acids leads to inefficient triglyceride digestion, then intestinal chylomicron formation will be reduced, not elevated (due to reduction of lipid uptake into the enterocyte). Transport of the water soluble B vitamins into the intestinal cells is not dependent on lipid digestion, as is fat-soluble vitamin absorption. The conjugation of bile acids will not affect the pH of the intestinal lumen, nor will it affect the secretion of zymogens from the pancreas to the intestine. The reactions involved in the conjugation of the bile acids are shown below.

A knockout mouse was created in which the ability to create conjugated bile salts was greatly impaired. The net result of this mutation in a mouse fed a normal diet is which of the following? (A) Steatorrhea (B) Elevated levels of chylomicrons (C) Deficiency of B vitamins (D) Reduced pH in the intestinal lumen (E) Reduced secretion of pancreatic zymogens

*The answer is B.* The court-ordered medication is disulfiram. Disulfiram inhibits aldehyde dehydrogenase, which greatly reduces the amount of acetaldehyde that is converted to acetate. This causes an accumulation of acetaldehyde, which is the substance responsible for the symptoms of a "hangover," including nausea and vomiting. Alcohol dehydrogenase reduces ethanol to acetaldehyde. Acetyl-CoA synthetase converts acetate to acetyl-CoA.

A man has just received his fourth DUI citation. The judge orders an alcohol dependency program complete with a medication that makes him have nausea and vomiting if he drinks alcohol while taking the medication. The drug-induced illness is due to the buildup of which one of the following? (A) Ethanol (B) Acetaldehyde (C) Acetate (D) Acetyl-CoA (E) Acetyl phosphate

*The answer is D.* The patient has Tangier disease, which is a defect in the ATP-binding cassette protein 1 (ABC1), a transporter in cell membranes which allows cholesterol efflux from the membrane into the HDL particle. Once inside the HDL particle, the cholesterol is trapped through esterification into a cholesterol ester. The HDL particle can then return the cholesterol to the liver for further recycling. The defect in the patient is not in HMG-CoA reductase (required for the biosynthesis of cholesterol), the AMP-activated kinase (a regulator of HMG-CoA reductase), LCAT (lecithin-cholesterol acyltransferase, the enzyme which esterifies cholesterol in the HDL particle), or CETP (cholesterol ester transfer protein, a protein which exchanges HDL cholesterol esters for VLDL triglyceride).

A patient has enlarged orange tonsils, hepatosplenomegaly, loss of sensation in hands and feet, and clouding of the corneas. His HDL levels are 18 mg/dL. The molecular defect in this patient is present in which of the following proteins? (A) HMG-CoA reductase (B) AMP-activated protein kinase (C) Lecithin cholesterol acyltransferase (D) ABC1 (E) Cholesterol ester transfer protein

*The answer is C.* This patient has the classic symptoms of Guillain-Barré syndrome which is an inflammatory autoimmune neuritis wherein T-cells formed in response to a viral illness mistakenly attack the myelin sheath of peripheral nerves. The myelin sheaths are composed primarily of sphingolipids and phospholipids and do not contain high-density lipoproteins, elastin, glycogen, or a significant level of glycoprotein.

A patient presents with rapidly progressive weakness of the lower extremities, loss of deep tendon reflexes, respiratory distress, and autonomic dysfunction following a flulike illness. This disease is an autoimmune inflammatory reaction to tissue made up chiefly of which of the following chemical structures? (A) High-density lipoproteins (B) Elastin (C) Sphingolipids (D) Glycoproteins (E) Glycogen

*The answer is A.* Foods considered the highest sources of sphingolipids include dairy and soy products. Foods highest in phospholipids include those high in lecithin, such as eggs, soy, and wheat. Sphingolipids and phospholipids are found mostly in neural tissue. Other organs and muscle do not contain as high a quantity of these lipids as do neural tissues.

A patient presents with rapidly progressive weakness of the lower extremities, loss of deep tendon reflexes, respiratory distress, and autonomic dysfunction following a flulike illness. This patient is in the recovery phase of her illness. She wants to "naturally" help her body recover using dietary methods. Which of the following foods is best in providing the chemicals needed to regrow the affected tissues? (A) Soybeans (B) Calves' liver (C) Pork kidney (D) Green leafy vegetables (E) Potatoes

*The answer is C.* Lipoprotein (a) is an LDL particle with a covalently linked apoprotein A (linked to apoprotein B100) attached to the particle. The presence of this unusual lipoprotein particle has been positively correlated with the presence of heart disease. The role of this particle is unknown, but may be related to coagulation, since apoprotein A resembles plasminogen in structure. Lp (a) levels do not regulate the levels of platelets in the circulation.

A patient sees his or her physician for continuing treatment of hypercholesterolemia. Recent blood work has indicated a substantial increase in the level of lipoprotein (a). Such a result would suggest which of the following? (A) Substantially reduced risk for cardiovascular complications (B) No change in risk for cardiovascular complications (C) Increased risk for cardiovascular complications (D) Increased platelet count (E) Decreased platelet count

*The answer is D.* Leukotrienes C4, D4, and E4 together compose the slow-reacting substance of anaphylaxis (SRS-A), which is thought to be the cause of asphyxiation in individuals not treated rapidly enough following an anaphylactic shock. SRS-A is up to 1000 times more effective than histamines in causing bronchial muscle constriction. Anti-inflammatory steroids are usually given intravenously to end chronic bronchoconstriction and hypotension following a shock. The steroids block phospholipase A2 action, preventing the synthesis of leukotrienes from arachidonic acid. Acute treatment involves epinephrine injected subcutaneously initially and then intravenously. Antihistamines such as diphenhydramine are administered intravenously or intramuscularly.

A patient stung by a bee is rushed into the emergency room with a variety of symptoms including increasing difficulty in breathing due to vasal and bronchial construction. While your subsequent treatment is to block the effects of histamine and other acute-phase reactants released by most cells, you must also block the slow-reacting substance of anaphylaxis (SRS-A), which is the most potent constrictor of the muscles enveloping the bronchial passages. What is SRS-A composed of? a. Thromboxanes b. Interleukins c. Complement d. Leukotrienes e. Prostaglandins

*The answer is A.* These are the clinical features of lipoprotein lipase deficiency (type I lipoproteinemia). LDL receptor defects would result in elevated LDLs. HMG-CoA reductase and ApoB-100 have no direct relationship to chylomicrons. ApoB-48 deficiency would result in decreased production of chylomicrons.

A patient with a history of recurring attacks of pancreatitis, eruptive xanthomas, and increased plasma triglyceride levels (2,000 mg/dl) associated with chylomicrons, most likely has a deficiency in A. lipoprotein lipase B. LDL receptors C. HMG-CoA reductase D. apoB-48 E. apoB-100 receptor

*The answer is E.* The patient is heterozygous for a mutation in the LDL receptor (familial hypercholesterolemia). This condition leads to elevated LDL levels since there are insufficient receptors available to remove LDL from the circulation. If left untreated, heart attacks are common in such patients before the age of 50. This condition is treated with statins, which reduce endogenous cholesterol synthesis, thereby leading to an upregulation of LDL receptors, which allows for normal LDL uptake from the circulation. Mutations in LCAT (familial LCAT defi ciency) are rare and do not often lead to premature atherosclerotic disease (although some exceptions are noted), but do lead to kidney and corneal damage due to large amounts of unesterified cholesterol present in those tissues. HDL level in these individuals is usually less than 10 mg/dL, which is not observed in our patient. Mutations in CETP (cholesterol ester transfer protein) lead to elevations in HDL levels and would not be responsive to statin action. Mutations in ABC1 lead to Tangier disease, which would lead to a reduction in HDL levels, which is not seen in this patient. A deficiency in apoB100 would impair VLDL synthesis and would actually reduce circulating LDL levels since there is less VLDL present to be converted to LDL.

A patient, 45-year-old male, BMI of 25, has had a history of elevated cholesterol (~300 mg/dL), with normal triglyceride levels (~125 mg /dL), and HDL levels (48 mg /dL). Treatment with statins has reduced his serum cholesterol to 180 mg /dL. The patient's father had a similar history and died of a heart attack at age 48. A potential mutation in this patient would be in which of the following proteins? (A) LCAT (B) CETP (C) ABC1 (D) Apo B100 (E) LDL receptor

*The answer is E.* Prostaglandins are synthesized from arachidonic acid. Arachidonic acid is synthesized from linoleic acid, an essential fatty acid obtained by humans from dietary lipids. The teenager would be able to synthesize all other compounds, but presumably in somewhat depressed amounts.

A teenager, concerned about his weight, attempts to maintain a fat-free diet for a period of several weeks. If his ability to synthesize various lipids were examined, he would be found to be most deficient in his ability to synthesize: A. triacylglycerol. B. phospholipids. C. cholesterol. D. sphingolipids. E. prostaglandins.

*The answer is D.* Insulin release stimulates the secretion of lipoprotein lipase (LPL) from fat and muscle cells such that the capillaries infiltrating these tissues have the lipase bound to extracellular matrix material. Then, as the triglyceride-rich particles move through the tissues, they bind to LPL via apolipoprotein CII, and the triglyceride is digested and the fatty acids used by the tissues. In the absence of insulin, LPL levels are low, and the particles have a longer half-life in circulation due to the reduced rate of digestion, which contributes to hypertriglyceridemia. If there were reduced synthesis of VLDL, triglycerides in the circulation would be reduced, not increased. Insulin does not alter the rate of apolipoprotein CII production. The release of insulin decreases fatty acid oxidation (promoting fatty acid synthesis), but if increased fatty acid oxidation did occur, then triglycerides would not accumulate in the circulation. Insulin also does not alter the synthesis of apolipoprotein B100 in the liver, which is required for VLDL synthesis.

A type 1 diabetic who has neglected to take his insulin for a few days displays both hyperglycemia and hypertriglyceridemia. The hypertriglyceridemia is due, in part, to which of the following? (A) Reduced synthesis of VLDL (B) Reduced production of apolipoprotein CII (C) Increased fatty acid oxidation (D) Reduced secretion of LPL (E) Increased synthesis of B100

*The answer is B.* ApoB-48 is required for intestinal absorption of dietary fat in the form of chylomicrons. ApoB-100 formation is also impaired in these patients, but this would not explain the clinical symptoms described.

Abetalipoproteinemia is a genetic disorder characterized by malabsorption of dietary lipid, steatorrhea (fatty stools), accumulation of intestinal triglyceride, and hypolipoproteinemia. A deficiency in the production of which apoprotein would most likely account for this clinical presentation? A. ApoB-100 B. ApoB-48 C. ApoC-II D. ApoA-I E. ApoE

*The answer is C.* By having different phospholipid compositions of the inner and outer leaflets of membranes, one can utilize phospholipid head groups (which face the aqueous phase of their leaflet) as markers for "inside" and "outside" the membrane structure. For example, the exposure of phosphatidylserine on the "outside" of red blood cells as is seen in spur cell anemia is a signal for the removal of the cells from circulation by the spleen, as the serine residue should be facing the "inside" of the red blood cell. Spur cells are large red blood cells covered with spikelike projections from preferential overexpansion of outer membrane components, leading to a spurlike shape. Movement of the phospholipid is a signal of cell aging. The melting temperature of the membrane is better determined by the fatty acid composition of the phospholipids, not the head group composition. Not all phospholipids are represented in all membranes (for example, cardiolipin is found almost exclusively in the mitochondria). Assymetric phospholipid compositions do not distinguish one organelle from another (that is primarily due to protein content), and assymetry in phospholipid composition may promote fusion (vesicles need to bud from and fuse with other membranes, particularly in the Golgi apparatus).

An alcoholic patient with advanced cirrhosis presents with spur cell anemia. For virtually all cell types and organelles, the phospholipid composition of the inner and outer leaflets of the membrane is different. The spur cell anemia is the result of the loss of one potential benefit of such phospholipid asymmetry. Which of the following best explains this benefit? (A) To vary the melting temperature of the membrane (B) To represent all phospholipids species within the membrane (C) To mark cells for recognition by outside systems (D) To distinguish between intracellular organelles (E) To prevent fusion of intracellular organelles

*The answer is D.* Dipalmitoylphos phatidylcholine (DPPC, or dipalmitoyl lecithin) is the lung surfactant found in mature, healthy lungs. RDS can occur in lungs that make too little of this compound. If the lecithin/sphingomyelin ratio in amniotic is greater than two, a newborn's lungs are considered to be sufficiently mature—premature lungs would be expected to have a ratio lower than two. The RDS would not be due to too few Type II pneumocytes—these cells would simply be secreting sphingomyelin rather than DPPC at 28 weeks of gestation.

An infant, born at 28 weeks of gestation, rapidly gave evidence of respiratory distress. Lab and x-ray results supported the diagnosis of infant respiratory distress syndrome (RDS). Which of the following statements about this syndrome is true? A. It is unrelated to the baby's premature birth. B. It is a consequence of too few Type II pneumocytes. C. The lecithin/sphingomyelin ratio in the amniotic fluid is likely to be greater than two. D. The concentration of dipalmitoylphosphatidylcholine in the amniotic fluid would be expected to be lower than that of a full-term baby. E. RDS is an easily treated disorder with low mortality.

*The answer is C.* During the synthesis of prostaglandins, a specific fatty acid is released from the 2′ position of membrane phospholipids by the action of phospholipase A2. After its release, the fatty acid can enter either the lipoxygenase pathway, which produces acid with an unknown biologic function, or the prostaglandin cyclooxygenase (also called prostaglandin synthetase) pathway. In the formation of prostaglandins from fatty acids, cyclooxygenase catalyzes formation of a cyclopentane ring and the introduction of three oxygen atoms. The type of prostaglandin produced depends on the starting fatty acid, which is always a derivative of an essential fatty acid. Eicosatrienoic acid yields series 1 prostaglandins, eicosatetraenoic (arachidonic) acid yields series 2 prostaglandins, and eicosapentaenoic acid yields series 3 prostaglandins. Aspirin, like indomethacin, decreases prostaglandin synthesis by inhibiting the oxygenase activity of cyclooxygenase.

Aspirin inhibits which of the following enzymes? a. Lipoprotein lipase b. Lipoxygenase c. Cyclooxygenase d. Phospholipase D e. Phospholipase A2

*The answer is B.* NSAIDs inhibit COX but not lipoxygenase, so any arachidonic acid available is used for the synthesis of bronchoconstricting-leukotrienes. NSAIDs have no effect on the CFTR protein, defects in which are the cause of cystic fibrosis. Steroids, not NSAIDs, inhibit phospholipase A2. COX is inhibited by NSAIDs, not activated. NSAIDs have no effect on phospholipases.

Aspirin-induced asthma (AIA) is a severe reaction to nonsteroidal anti-inflammatory drugs (NSAIDs) characterized by bronchoconstriction 30 minutes to several hours after ingestion. It is seen in as many as 20% of adults. Which of the following statements best explains the symptoms seen in patients with AIA? A. NSAIDs inhibit the activity of the CFTR protein, resulting in thickened secretions that block airways. B. NSAIDs inhibit COX but not lipoxygenase, resulting in the flow of arachidonic acid to leukotriene synthesis. C. NSAIDs activate the COX activity of PGH synthase, resulting in increased synthesis of prostaglandins that promote vasodilation. D. NSAIDs activate phospholipases, resulting in decreased amounts of dipalmytoylphosphatidylcholine and alveolar collapse (atelectasis).

*The answer is B.* The most common sphingolipid in mammals is sphingomyelin. Ceramide, the basic structure from which all sphingolipids are derived, is composed of the 18-carbon sphingosine connected via its amino group to a fatty acid by an amide linkage. The fatty acid is usually long-chain (18 to 26 carbons) and is saturated or monosaturated. Except for the lack of the glycerol backbone, sphingolipids are quite similar in structure and physical properties to the phospholipids phosphatidylcholine and phosphatidylethanolamine. Either phosphoryl choline or phosphoryl ethanolamine is the head group attached to ceramide. If a neutral sugar residue is the polar head group attached to ceramide, a cerebroside is formed. If oligosaccharide head groups containing sialic acid are used, gangliosides are formed. All sphingolipids are important membrane constituents.

Ceramide is a precursor to which of the following compounds? a. Phosphatidyl serine b. Sphingomyelin c. Phosphatidyl glycerol d. Phosphatidyl choline e. Phosphatidyl ethanolamine

*The answer is B.* Chylomicrons are triglyceride-rich transport particles containing dietary lipids. Very-low-density lipoproteins (VLDLs) are triglyceride- and cholesterol-containing particles from the liver that contain endogenously packaged lipids. Delipidation of triglycerides from VLDLs leads to formation of intermediate forms (IDLs), and finally to a cholesterol-enriched small particle, the low-density lipoprotein (LDL). Thus, VLDL → IDL → LDL.

Chylomicrons, intermediate-density lipoproteins (IDLs), low density lipoproteins (LDLs), and very-low-density lipoproteins (VLDLs) are all serum lipoproteins. What is the correct ordering of these particles from the lowest to the highest density? a. LDLs, IDLs, VLDLs, chylomicrons b. Chylomicrons, VLDLs, IDLs, LDLs c. VLDLs, IDLs, LDLs, chylomicrons d. Chylomicrons, IDLs, VLDLs, LDLs e. LDLs, VLDLs, IDLs, chylomicrons

*The answer is B.* HDL is protective, in part, due to its ability to remove excess cholesterol from cell membranes and return it to the liver. In order to accomplish this, the cholesterol, after transport to the HDL particle via the participation of ABC1, needs to be trapped within the core of the HDL particle, and this is accomplished by esterification and converting the cholesterol to a cholesterol ester. LCAT (lecithin cholesterol acyl transferase) is the enzyme that creates a cholesterol ester. The reaction is the transfer of a fatty acid from phosphatidyl choline (lecithin) to cholesterol, creating the cholesterol ester and lysophosphatidyl choline. ACAT (acyl-CoA cholesterol acyl transferase) creates cholesterol esters in cells, but not in the HDL particles. CETP exchanges HDL cholesterol esters for VLDL triglyceride.

Current American Heart Association Guidelines indicate that an adult male should have HDL levels equal to or greater than 40 mg/dL. A necessary enzyme contributing to HDL's protective effect is which of the following? (A) CETP (B) LCAT (C) ACAT (D) AMP-activated protein kinase (E) Protein kinase A

*The answer is C.* During oxidation of fatty acids in the mitochondrial matrix, fatty acids are processed while being attached to the phosphopantetheine group of coenzyme A (CoA). Acyl carrier protein (ACP) is only involved as part of the fatty acid synthase multienzyme complex in the cytosol. The phosphopantetheine group of ACP binds malonic acid, releasing the CoA group of malonyl CoA. It then condenses malonic acid with an acetyl group derived from acetyl CoA in the first round of synthesis. In subsequent rounds, the growing acyl chain is condensed with malonyl CoA.

During fatty acid metabolism in humans, coenzyme A (CoA) is different from acyl carrier protein (ACP) in which one of the following ways? a. Binding of malonic acid with a phosphopantetheine b. Binding of fatty acids c. Function in fatty acid oxidation d. Function in the cytosol e. Function in fatty acid synthesis

*The answer is B.* HMG-CoA reductase is the rate-limiting enzyme in cholesterol biosynthesis. And if LDL does not bind to its receptor, cholesterol will not enter the cell to inhibit this enzyme.

Failure to bind LDL to its receptor results in uncontrolled synthesis of cholesterol. This occurs because synthesis of which of the following enzymes is not repressed? (A) Acyl CoA cholesterol acyltransferase (B) 3-Hydroxy-3-methylglutaryl (HMG)-CoA reductase (C) Lecithin-cholesterol acyltransferase (D) Lipoprotein lipase (E) Lysosomal protease

*The answer is C.* During lipolysis, triglycerides are split into three free fatty acids and glycerol. The free fatty acids, as well as the free glycerol, diffuse into the bloodstream, where they are circulated throughout the body. The free fatty acids are used as an energy source for many tissues, primarily muscle. The free glycerol that is released cannot be phosphorylated back to glycerol-3-phosphate in the adipose tissue because it lacks glycerol kinase. However, the free glycerol released in lipolysis is taken up by the liver, where it can be phosphorylated to glycerol-3-phosphate. The phosphorylated glycerol can enter glycolysis or gluconeogensesis at the level of triose phosphates. If gluconeogenesis occurs, for every 2 mol of glycerol-3 phosphate, 1 mol of glucose can be synthesized.

For every 2 mol of free glycerol released by lipolysis of triacylglycerides in adipose tissue a. 2 mol of triacylglycerides is released b. 2 mol of free fatty acids is released c. 1 mol of glucose can be synthesized in gluconeogenesis d. 1 mol of triacylglyceride is released e. 3 mol of acyl CoA is produced

*The answer is D.* With a competitive inhibitor, there will be an increase in Km with no change in Vmax. Choice A would be for a noncompetitive inhibitor (Vmax decreased, Km unaltered).

From a Lineweaver-Burk plot, the Km and Vmax of this rate-limiting en-zyme were calculated to be 4 10-3 M and 8 10² mmol/h, respectively. If the given experiment is repeated in the presence of simvastatin, which of the following values would be obtained?

*The answer is A.* Plasma membranes are unique as compared to intracellular membranes in that their composition contains cholesterol, glycoproteins, and glycolipids known as gangliosides. Plasma membranes of the cells of different tissues are distinguished from each other due to the properties that make them unique. Hormone receptors allow each cell type to respond to systemic stimulation appropriately. All chronic hormone receptors are localized to plasma membranes and upon stimulation release a second messenger into the interior of the cell. Glucagon, like epinephrine and norepinephrine, stimulates adenylate cyclase to produce cyclic AMP. Glucagon is found on the plasma membranes of liver and adipose tissue cells.

Gangliosides and receptors for hormones such as glucagon can be found in which of the diagrammatic structures shown? a. Structure A b. Structure B c. Structure C d. Structure D e. Structure E

*The answer is E.* In mammals, a variety of fatty acids are considered essential and cannot be synthesized. These include linoleate (C-18 cis-Δ9, Δ12) and linolenate (C-18 cis-Δ9, Δ12, Δ15). Either these fatty acids or fatty acids for which they are precursors must be supplied in the diet as starting points for synthesis of a variety of other unsaturated fatty acids that lead to the synthesis of prostaglandins, thromboxanes, and leukotrienes. For example, arachidonate, a 20-carbon fatty acid with four double bonds, is derived from linolenate. Arachidonate gives rise to some prostaglandins, thromboxanes, and leukotrienes. Some fatty acids must be obtained in the diet because of the limitations governing enzymes of fatty acid synthesis in humans; that is, double bonds cannot be introduced beyond the 9-10 bond position of carbons in the fatty acid chain, and subsequent double bonds after the first must be separated by two single bonds. Thus, linolenate and linoleate cannot be synthesized in humans.

In humans, the formation of the fatty acid C-18-Δ9,Δ12 can be derived from which of the following? a. C-18 cis-Δ9 b. C-18 cis-Δ6 c. C-18 d. C-16 cis-Δ6,Δ9 e. C-18 cis-Δ9,Δ12

*The answer is D.* In vitamin B12 deficiency, methlmalony-CoA mutase is unable to convert methylmalonyl to succinyl-CoA.

Inhibition of which of the following enzymes is responsible for the peripheral neuropathy involved in vitamin B12 deficiency? A. Carnitine acyltransferase-1 B. Carnitine acyltransferase-2 C. Medium chain acyl-CoA dehydrogenase D. Methylmalonyl-CoA mutase E. Propionyl-CoA carboxylase

*The answer is D.* Infants placed on chronic low-fat formula diets often develop skin problems, impaired lipid transport, and eventually poor growth. This can be overcome by including linoleic acid to make up 1 to 2% of the total caloric requirement. Essential fatty acids are required because humans have only Δ4, Δ5, Δ6, and Δ9 fatty acid desaturase. Only plants have desaturase greater than Δ9. Consequently, certain fatty acids such as arachidonic acid cannot be made "from scratch" (de novo) in humans and other mammals. However, linoleic acid, which plants make, can be converted to arachidonic acid. Arachidonate and eicosapentaenoate are 20-carbon prostanoic acids that are the starting point of the synthesis of prostaglandins, thromboxanes, and leukotrienes.

It has been noted that infants placed on extremely low-fat diets for a variety of reasons often develop skin problems and other symptoms. This is most often due to a. Lactose intolerance b. Glycogen storage diseases c. Antibody abnormalities d. Deficiency of fatty acid desaturase greater than Δ9 e. Deficiency of chylomicron and VLDL production

*The answer is A.* Gangliosides are continually synthesized and broken down. The specific hydrolases that degrade gangliosides by sequentially removing terminal sugars are found in lysosomes. In the lipid storage disease known as Tay-Sachs disease, ganglioside GM2 accumulates because of a deficiency of β-N-acetylhexosaminidase, a lysosomal enzyme that removes the terminal N-acetylgalactosamine residue. Homozygotes produce virtually no functional enzyme and suffer weakness, retardation, and blindness. Death usually occurs before infants are 3 years old. Carriers (heterozygotes) of the autosomal recessive disease produce approximately 50% of the normal levels of enzyme but show no ill effects. In high-risk populations, such as Ashkenazi Jews, screening for carrier status may be performed.

Leukocyte samples isolated from the blood of a newborn infant are homogenized and incubated with ganglioside GM2. Approximately 47% of the expected normal amount of N-acetylgalactosamine is liberated during the incubation period. These results indicate that the infant a. Is a heterozygote (carrier) for Tay-Sachs disease b. Is homozygous for Tay-Sachs disease c. Has Tay-Sachs syndrome d. Will most likely have mental deficiency e. Has relatively normal β-N-acetylhexosaminidase activity

*The answer is B.* Fatty acid oxidation increases the levels of acetyl CoA within the mitochondrial matrix, and acetyl-CoA is a potent activator of pyruvate carboxylase, a key gluconeogenic enzyme (it will convert pyruvate to oxaloacetate, a necessary first step to bypass the irreversible pyruvate kinase reaction). Acetyl-CoA cannot be used to synthesize net glucose, so it is not an effective precursor of glucose production. Acetyl-CoA does not activate PEP carboxykinase (that enzyme is transcriptionally controlled), nor does it affect pyruvate kinase (a cytoplasmic enzyme). PFK-2 is not regulated by acetyl-CoA (phosphorylation by protein kinase A is the key regulator effect for PFK-2 in the liver).

Liver fatty acid oxidation leads to an enhancement of gluconeogenesis via which of the following? (A) Generation of precursors for glucose synthesis (B) Activation of pyruvate carboxylase (C) Activation of phosphoenolpyruvate carboxykinase (D) Inhibition of pyruvate kinase (E) Inhibition of PFK-2

*The answer is C.* The macrophages take up oxidized LDL using a scavenger receptor, SR-A1, which is not downregulated. This allows the receptor to remain on the cell surface and to constantly import oxidized LDL into the cell. The high levels of cholesterol in the foam cells is not due to a change in activity of ACAT or LCAT (which is found in HDL particles), nor is there upregulation of HMG-CoA reductase (which would produce more endogenous cholesterol, which is unlikely since the cell is filled with cholesterol and cholesterol esters). Macrophages do not use the LDL receptor for importing oxidized LDL.

Macrophages found in arterial fatty streaks are often lipid filled and become foam cells. Such large amounts of cholesterol uptake into these cells is possible due to which of the following? (A) Increased activity of ACAT within the foam cell (B) Increased activity of LCAT within the foam cell (C) Constant SR-A1 expression on the cell surface (D) Upregulation of HMG-CoA reductase (E) Increased activity of the LDL receptor

*The answer is D.* Under fasting conditions, the total cholesterol measured will be the sum of the cholesterol in the HDL particles, the LDL particles, and VLDL. Chylomicrons should be nil under fasting conditions. The total cholesterol is measured, as are HDL and triglycerides. Since the VLDL is the primary triglyceride carrier under these conditions, the cholesterol content of the VLDL is estimated to be 20% that of the triglyceride content. Thus, the formula for calculating LDL values is LDL = total cholesterol - HDL - [(TG)/5].

Many clinical labs report lipid values using a calculated value for LDL. This calculation estimates the cholesterol content in which of the following particles under fasting conditions? (A) HDL (B) LDL (C) IDL (D) VLDL (E) Chylomicron

*The answer is A.* Multiple sclerosis is a demyelination disease characterized by chronic inflammation. The primary losses are of the phospholipids and sphingolipids composing myelin membrane sheets of nerves in white matter. Consequently, brain white matter eventually looks like gray matter. High levels of sphingolipids and phospholipids are observed in the cerebrospinal fluid.

Multiple sclerosis is a disease characterized by chronic inflammation. There are significant data to indicate that susceptibility to multiple sclerosis is inherited and causes a primary change in a. Membrane lipids b. Anti-inflammatory steroids c. Blood proteins d. Stored carbohydrates e. Nucleotide metabolism

*The answer is A.* The sources of NADPH for synthesis of fatty acids are the pentose phosphate pathway and cytosolic malate formed during the transfer of acetyl groups to the cytosol as citrate. The enzyme citrate lyase splits citrate into acetyl CoA and oxaloacetate. The oxaloacetate is reduced to malate by NADH. NADP-linked malate enzyme catalyzes the oxidative decarboxylation of malate to pyruvate and carbon dioxide. Thus, the diffusion of excess citrate from the mitochondria to the cytoplasm of cells not only provides acetyl CoA for synthesis of fatty acids but NADPH as well. One NADPH is produced for each acetyl CoA produced. However, most of the NADPHs needed for synthesis of fatty acids are derived from the pentose phosphate pathway. For this reason, adipose tissue has an extremely active pentose phosphate pathway.

Most of the reducing equivalents utilized for synthesis of fatty acids can be generated from a. The pentose phosphate pathway b. Glycolysis c. The citric acid cycle d. Mitochondrial malate dehydrogenase e. Citrate lyase

*The answer is E.* The genetic defect would result in malabsorption of the 3 fatty acids listed, but only linoleate is strictly essential in the diet. Absorption of water-soluble ascorbate and folate would not be significantly affected.

Patients with abetalipoproteinemia exhibit membrane abnormalities in their erythrocytes with production of acanthocytes (thorny-appearing cells). This unusual red cell morphology would most likely result from malabsorption of A. palmitic acid B. ascorbic acid C. arachidonic acid D. folic acid E. linoleic acid

*The answer is C.* Phosphatidylinositol is used as the substrate to provide signaling molecules in response to the appropriate stimuli (the phosphatidylinositol cycle). As such, it must face the cytoplasm of the cell such that when the inositol phosphate derivatives are produced, such as IP3, they can move to their target receptors to elicit a cellular response. Inositol contains six hydroxyl groups and is a very hydrophilic molecule. Inositol's structure is quite different from glucose (there is no carbonyl group in inositol), so it is unlikely that glucose and inositol would compete for binding to the same receptors. Phosphatidylinositol does not bind to phosphatidylserine in the inner leafl et of membranes. Inositol also does not interact with the actin cytoskeleton.

Phosphatidylinositol contributes to phospholipid bilayer asymmetry by being in the inner leaflet of membranes, facing the cytoplasm of the cell. This is most likely due to which of the following? (A) The hydrophobic nature of inositol is unstable facing the cellular exterior (B) Inositol is very similar in structure to glucose and could compete with glucose for binding of ligands to the extracellular surface (C) Phosphatidylinositol acts as a substrate for intracellular processes (D) Phosphatidylinositol binds to phosphatidylserine, another inner leaflet specific phospholipid (E) Inositol interacts with intracellular actin, linking the inner leaflet to a cell's cytoskeleton

*The answer is D.* Ethanol is both water- and lipid-soluble. It is easily absorbed from the gastrointestinal tract and is distributed throughout the body via the blood stream. It is mostly metabolized in the liver, so the level would be high in this tissue. It is lipid-soluble, so it would be found in fatty tissue. It has many central effects in the brain, so it easily passes the blood- brain barrier. The central cornea has no arterial supply. The only way alcohol could accumulate in the central cornea would be through diffusion into the aqueous humor and then into the central cornea, a slower and less-efficient system. Therefore, the tissue with the lowest level of alcohol would be the central cornea.

Remembering the distribution and solubility of ethanol, after several drinks with an evening meal, in which of the following tissues would you find the LEAST amount of alcohol? (A) Brain (B) Liver (C) Fatty tissue (D) Central cornea

*The answer is C.* Statins are effective in lowering circulating cholesterol levels due to a series of events. First, the statins inhibit HMG-CoA reductase, reducing intracellular synthesis of cholesterol. The reduced cholesterol levels in the cell then upregulate the synthesis of LDL receptors, which remove LDL from circulation, thereby reducing circulating cholesterol levels. Familial hypercholesterolemia (FH) is a mutation in the LDL receptor, making the receptor unable to bind LDL. In homozygous familial hypercholesterolemia, both LDL receptor genes are mutated, and the LDL receptors are nonfunctional. Upregulating nonfunctional LDL receptors will not lead to a reduction of LDL in the circulation, so such individuals are resistant to statin action. FH is not due to a resistant HMG-CoA reductase, nor an inability of statins to reach their target. FH is not related to reverse cholesterol transport, nor to LCAT.

Statins are ineffective in lowering cholesterol levels in individuals with homozygous familial hypercholesterolemia due to which of the following? (A) HMG-CoA reductase is resistant to statins (B) Statins cannot enter the liver cells (C) LDL receptors are nonfunctional (D) Reverse cholesterol transport is inoperative in these patients (E) LCAT is resistant to statin action

*The answer is D.* Must know that mevalonate precedes squalene and lanosterol in the pathway, and that methylmalonate and acetoacetate are not associated with cholesterolgenesis.

The anticholesterolemic action of simvastatin is based on its effectiveness as a competitive inhibitor of the rate-limiting enzyme in cholesterol biosynthesis. The reaction product normally produced by this enzyme is A. squalene B. methylmalonate C. lanosterol D. mevalonate E. acetoacetate

*The answer is C.* Fatty acid synthesis in the cytosol terminates at palmitate, a C16 saturated fatty acid. Elongation of acyl groups can occur from palmitate as well as from other dietary saturated and unsaturated fatty acids of lengths C10 and greater. Longer-chain elongation of fatty acids occurs in the endoplasmic reticulum (structure C) using malonyl CoA as the acetyl donor and NADPH as the reductant. Very-long-chain and long-chain fatty acids are preferentially catabolized in peroxisomes (structure J). Other structures diagrammed in the figure are the plasma membrane (A), mitochondrion (B), nucleoplasm (D), nucleolus (E), Golgi apparatus (F), secretory vesicles (G), caveolae such as those taking up low-density lipoprotein from its receptor (H), and lysosomes (I).

The elongation of fatty acids occurs in which of the diagrammatic structures shown below? a. Structure A b. Structure B c. Structure C d. Structure D e. Structure E

*The answer is D.* In humans, the end product of fatty acid synthesis in the cytosol is palmitic acid. The specificity of cytosolic multienzyme, single-protein fatty acid synthetase is such that once the C16 chain length is reached, a thioesterase clips off the fatty acid. Elongation as well as desaturation of de novo palmitate and fatty acids obtained from the diet occur by the action of enzymes in the membranes of the endoplasmic reticulum.

The end product of cytosol fatty acid synthetase in humans is a. Oleic acid b. Arachidonic acid c. Linoleic acid d. Palmitic acid e. Palmitoleic acid

*The answer is E.* The fatty acid synthase complex of mammals is composed of two identical subunits. Each of the subunits is a multienzyme complex of seven enzymes and the acyl carrier protein component. All the components are covalently linked together; thus, all the components are on a single polypeptide chain, which functions in the presence of another identical polypeptide chain. Each cycle of fatty acid synthesis employs the acyl carrier protein and six enzymes: acetyl transferase, malonyl transferase, β-ketoacyl synthase, β-ketoacyl reductase, dehydratase, and enoyl reductase. When the final fatty acid length is reached (usually C16), thioesterase hydrolyzes the fatty acid off of the synthase complex.

The fatty acid synthase complex of mammals a. Is a dimer of unsimilar subunits b. Is composed of seven different proteins c. Dissociates into eight different proteins d. Catalyzes eight different enzymatic steps e. Is composed of covalently linked enzymes

*The answer is B.* The ketone bodies, β-hydroxybutyrate and acetoacetate, are synthesized in liver mitochondria from acetyl CoA. The liver produces ketone bodies under conditions of fasting associated with high rates of fatty acid oxidation. Higher amounts of β-hydroxybutyrate than acetoacetate are produced, since high liver levels of NADH lead to the dehydrogenation of acetoacetate.

The formation of β-hydroxybutyrate occurs in which of them diagrammatic structures shown? a. Structure A b. Structure B c. Structure C d. Structure D e. Structure E

*The answer is B.* The uptake of exogenous cholesterol by cells results in a marked suppression of endogenous cholesterol synthesis. Low-density human lipoprotein not only contains the greatest ratio of bound cholesterol to protein but also has the greatest potency in suppressing endogenous cholesterogenesis. LDLs normally suppress cholesterol synthesis by binding to a specific membrane receptor that mediates inhibition of hydroxymethylglutaryl (HMG) coenzyme A reductase. In familial hypercholesterolemia, the LDL receptor is dysfunctional, with the result that cholesterol synthesis is less responsive to plasma cholesterol levels. Suppression of HMG CoA reductase is attained using inhibitors (statins) that mimic the structure of mevalonic acid, the natural feedback inhibitor of the enzyme.

The major source of extracellular cholesterol for human tissues is a. Very-low-density lipoproteins (VLDLs) b. Low-density lipoproteins (LDLs) c. High-density lipoproteins (HDLs) d. Albumin e. γ-globulin

*The answer is E.* In normal persons, plasma cholesterol levels average about 175 mg/dL. The level is about 300 mg/dL in heterozygotes with the autosomal dominant genotype and about 680 mg/dL in homozygotes. The single mutagenic defect results in a functional loss of low-density lipoprotein (LDL) receptors on the plasma membranes of cells other than those of the liver. This prevents the normal clearing of LDLs from the blood plasma by endocytosis.

The primary biochemical lesion in homozygotes with familial hypercholesterolemia (type IIa) is a. The loss of feedback inhibition of liver hydroxymethylglutaryl CoA reductase b. The increased production of low-density lipoproteins from very-low-density lipoproteins c. The loss of apolipoprotein B d. The malfunctioning of acyl CoA-cholesterol acyl transferase (ACAT) e. The functional deficiency of plasma membrane receptors for low-density lipoproteins

*The answer is D.* The synthesis of 3-hydroxy-3-methylglutaryl CoA requires the condensation of three acetyl CoA groups. The two enzymatic steps involved are the first two steps of cholesterol synthesis and ketone body synthesis. While cholesterol synthesis occurs in the cytosol of most mammalian tissues, ketone body synthesis can only occur in the mitochondria of liver cells. Not only are cholesterol synthesis and ketone body synthesis separated by compartmentalization, they are separated by metabolic needs. Cholesterol synthesis is an anabolic pathway that takes place when acetyl CoA production from excess dietary precursors is possible. In contrast, ketone body production by the liver occurs when acetyl CoA levels from β oxidation are high. This catabolic situation exists during fasting, starvation, and uncontrolled diabetes.

The synthesis of 3-hydroxy-3-methylglutaryl CoA can occur a. Only in mitochondria of all mammalian tissues b. Only in the cytosol of all mammalian tissues c. Only in the endoplasmic reticulum of all mammalian tissues d. In both the cytosol and mitochondria e. In lysosomes

*The answer is D.* Hormone-sensitive lipase is phosphorylated by cAMP-activated protein kinase, which is itself activated by epinephrine or glucagon. Fatty acids released from adipose tissue are carried in the plasma by serum albumin, not VLDL. During a fast, the amount of circulating triacylglycerol (found in chylomicrons and VLDL) will be low. Therefore, there is little substrate for lipoprotein lipase. The glycerol produced during triacylglycerol degradation cannot be metabolized by adipocytes or fibroblasts, but rather must go to the liver where it can be phosphorylated (by glycerol kinase).

Triacylglycerol molecules stored in adipose tissue represent the major reserve of substrate providing energy during a prolonged fast. During such a fast: A. the stored fatty acids are released from adipose tissue into the plasma as components of the serum lipoprotein particle, VLDL. B. free fatty acids are produced at a high rate in the plasma by the action of lipoprotein lipase on chylomicrons. C. glycerol produced by the degradation of triacylglycerol is an important direct source of energy for adipocytes and fibroblasts. D. hormone-sensitive lipase is phosphorylated and activated by a cAMP-activated protein kinase.

*The answer is A.* Only vitamin E is an antioxidant.

Uncontrolled phagocytosis of oxidized LDL particles is a major stimulus for the development of foam cells and fatty streaks in the vascular subendothelium. This process may be inhibited by increased dietary intake of A. vitamin E B. vitamin B6 C. vitamin D D. vitamin B12 E. vitamin K

*The answer is B.* Citrate is a potent activator of acetyl-CoA carboxylase for fatty acid synthesis.

What is the most positive activator of the process shown below? 8 Acetyl-CoA + n ATP + 14 NADPH → palmitate + 8 CoASH + nADP + nPi + 14 NADP A. Acetyl-CoA B. Citrate C. Malonyl-CoA D. Malate E. Oxaloacetate

*The answer is E.* To reform triglycerides from the incoming fatty acids, glycerol 3-P must be avail-able. The adipose can produce this only from DHAP in glycolysis.

When adipose tissue stores triglyceride arriving from the liver or intestine, glycolysis must also occur in the adipocyte. Which of the following products or intermediates of glycolysis is required for fat storage? A. Glycerol B. Glucose 6-phosphate C. Pyruvate D. Acetyl-CoA E. Dihydroxyacetone phosphate

*The answer is A.* Under conditions of active synthesis of fatty acids in the cytosol of hepatocytes, levels of malonyl CoA are high. Malonyl CoA is the activated source of two carbon units for fatty acid synthesis. Malonyl CoA inhibits carnitine acyltransferase I, which is located on the cytosolic face of the inner mitochondrial membrane. Thus, long-chain fatty acyl CoA units cannot be transported into mitochondria where β oxidation occurs, and translocation from cytosol to mitochondrial matrix is prevented. In this situation compartmentalization of membranes as well as inhibition of enzymes comes into play.

When the liver is actively synthesizing fatty acids, a concomitant decrease in β oxidation of fatty acids is due to a. Inhibition of a translocation between cellular compartments b. Inhibition by an end product c. Activation of an enzyme d. Detergent effects e. Decreases in adipocyte lipolysis

*The answer is B.* The fluid mosaic model of membrane structure shown in the question describes plasma membranes as a mosaic of globular proteins in a phospholipid bilayer. The lipids as well as the proteins are in a fluid and dynamic state capable of translational (side-toside) movement, but not "flip flop"-type movements. Hence, both the phospholipid and protein components are amphipathic, with a highly polar end in contact with the aqueous phase and hydrophobic residues buried within the membrane. Integral proteins may be embedded in the membrane or exposed on only one side, or they may extend completely through the membrane with different portions of the proteins exposed to opposite sides of the membrane. In contrast to the fluid mosaic model, the models of protein-coated bimolecular layer of lipid diagrammed in A and the unit membrane "railroad track" shown in D (which is based on osmium tetroxide fixed membranes) suggest that membranes are simply bimolecular layers of lipid coated with protein that does not penetrate the lipid. A simple bimolecular layer of lipid is shown in C, and a micelle of lipids is diagrammatically illustrated in E.

Which of the diagrammatic structures shown below represents the model of biologic membranes that most successfully accounts for membrane asymmetry? a. Structure A b. Structure B c. Structure C d. Structure D e. Structure E

*The answer is E.* The process of emulsification of hydrophobic fat globules by the detergent action of phospholipids and bile acids in the gut breaks the globules down to mixed micelles. The formation of small micelles from large fat globules greatly increases the surface area available for the action of hydrolytic lipases in the gut. Mixed micelle formation is dependent upon the amphipathic properties of bile acids and phospholipids that allow them to act as detergents. Simply put, the hydrophobic moieties (fatty acid chains) of phospholipids are inserted into the hydrophobic fat globules and the hydrophilic polar head groups interact with and face the water, in essence forming a monomolecular layer around the fat (triacylglycerides). This successful strategy of mixed micelles is used to solve many potential problems, including the transport of blood lipoproteins.

Which of the diagrammatic structures shown most clearly represents a model of the configuration of lipids obtained during the emulsification process that precedes hydrolysis during digestion? a. Structure A b. Structure B c. Structure C d. Structure D e. Structure E

*The answer is B.* A deficiency in carnitine, carnitine acyltransferase I, carnitine acyltransferase II, or acylcarnitine translocase can lead to an inability to oxidize long-chain fatty acids. This occurs because all of these components are needed to translocate activated long-chain (>10 carbons long) fatty acyl CoA across mitochondrial inner membrane into the matrix where β oxidation takes place. Once long-chain fatty acids are coupled to the sulfur atom of CoA on the outer mitochondrial membrane, they can be transferred to carnitine by the enzyme carnitine acyltransferase I, which is located on the cytosolic side of the inner mitochondrial membrane. Acylcarnitine is transferred across the inner membrane to the matrix surface by translocase. At this point the acyl group is reattached to a CoA sulfhydryl by the carnitine acyltransferase II located on the matrix face of the inner mitochondrial membrane.

Which of the following diseases reflects the loss of ability to move specific molecules between membrane-separated cellular compartments? a. McArdle's phosphorylase disease b. Carnitine deficiency c. Methanol poisoning d. Ethylene glycol poisoning e. Diphtheria

*The answer is E.* The enzyme phosphatidate phosphatase converts phosphatidic acid to diacylglycerol during synthesis of triacylglycerides. The function of adipose tissue is the storage of fatty acids as triacylglycerols in times of plenty and the release of fatty acids during times of fasting or starvation. Fatty acids taken in by adipocytes are stored by esterification to glycerol-3-phosphate. Glycerol-3-phosphate is derived almost entirely from the glycolytic intermediate dihydroxyacetone phosphate through the action of glycerol-3-phosphate dehydrogenase. Glycolytic enzymes are active in adipocytes during triglyceride synthesis, but those of glycogen degradation (low levels in adipocytes) and gluconeogenesis (i.e., glucose-6-phosphatase) are not. Glycerol kinase is not present to any great extent in adipocytes, so that glycerol freed during lipolysis is not used to reesterify the fatty acids being released. The enzyme triacylglyceride lipase is turned on by phosphorylation by a cyclic AMP-dependent protein kinase following epinephrine stimulation.

Which of the following enzymes is active in adipocytes following a heavy meal? a. Glycogen phosphorylase b. Glycerol kinase c. Hormone-sensitive triacylglyceride lipase d. Glucose-6-phosphatase e. Phosphatidate phosphatase

*The answer is A.* In mammals, β-carotene is the precursor of retinal, which is the basic chromophore of all visual pigments. Isopentenyl pyrophosphate and dimethylallyl pyrophosphate are isoprenoid isomers formed from the repeated condensation of acetyl CoA units. By continued condensation in mammalian systems, cholesterol can be formed. In plant systems, carotenoids are formed. In addition to producing the color of tomatoes and carrots, carotenoids serve as the light-absorbing molecules of photosynthesis. Ketone bodies are derived from condensation of acetyl CoA units but not from isoprenoid units. Vitamin C (ascorbic acid), carnitine, and thiamine (vitamin B1) are not derived from isoprenoid units.

Which of the following involves isoprenoids? a. The chromophore of visual pigments b. Carnitine c. Vitamin C d. Thiamine e. Ketone bodies

*The answer is B.* The essential fatty acid linoleic acid, with 18 carbons and two double bonds at carbons 9 and 18 (C-18:2-Δ9,12) is desaturated to form α-linolenic acid (C-18:3-Δ6,9,12), which is sequentially elongated and desaturated to form eicosatrienoic acid (C-20:3-Δ8,11,14) and arachidonic acid (C-20:4-Δ5,8,11,14), respectively. Many of the eicosanoids (20-carbon compounds)—prostaglandins, thromboxanes, and leukotrienes—are derived from arachidonic acid. The scientific name of arachidonic acid is eicosatetraenoic acid. Arachidonic acid can only besynthesized from essential fatty acids obtained from the diet. Palmitic acid (C-16:0) and oleic acid (C-18:1-Δ9) can be synthesized by the tissues.

Which of the following is an essential fatty acid? a. Palmitic acid b. Linoleic acid c. Arachidonic acid d. Oleic acid e. Eicosatetraenoic acid

*The answer is E.* Sphingolipidoses are lipid storage diseases that exhibit Mendelian (autosomal or X-linked recessive) inheritance. To date, all lipidoses studied demonstrate accumulation of a ceramide-containing sphingolipid due to the genetic deficiency of a specific hydrolytic enzyme involved in the breakdown of the sphingolipid in question. This leads to the accumulation of the sphingolipid because its synthetic rate is normal. Since the decrease in activity of the abnormal hydrolytic enzyme is similar in all tissues, diagnostic tests measuring the enzyme can easily be set up using skin biopsies or blood cell measurements. Heterozygous carriers can be screened and receive genetic counseling. Examples of sphingolipidoses include Tay-Sachs disease, Sandhoff's disease, and Gaucher's disease.

Which of the following is most characteristic of a sphingolipidosis? a. Multifactorial inheritance b. Variable activities of abnormal enzyme in different patient tissues c. Deficiency of a hydrolytic enzyme d. Abnormalities of sphingolipid synthesis e. Accumulation of ceramide-containing lipids

*The answer is C.* Carnitine is required to transport fatty acids across the inner mitochondrial membrane.

Which of the following is required to transport fatty acids across the inner mitochondrial membrane? (A) Acyl carrier protein (B) Albumin (C) Carnitine (D) Chylomicrons (E) Creatinine (F) Lecithin-cholesterol acyltransferase

*The answer is B.* In the postabsorptional (postprandial) state, plasma contains all the lipoproteins: chylomicrons derived from dietary lipids packaged in the intestinal epithelial cells and their remnants; very-low-density lipoproteins (VLDLs), which contain endogenous lipids and cholesterol packaged in the liver; low-density lipoproteins (LDLs), which are end products of delipidation of VLDLs; and high-density lipoprotins (HDLs), which are synthesized in the liver. HDLs are in part catalytic, since transfer of their CII apolipoprotein to VLDLs or chylomicrons activates lipoprotein lipase. In normal patients, only LDLs and HDLs remain in plasma following a 12-h fast, since both chylomicrons and VLDLs have been delipidated. Most of the cholesterol measured in blood plasma at this time is present in the cholesterol-rich LDLs. However, HDL cholesterol also contributes to the measurement.

Which of the following lipoproteins would contribute to a measurement of plasma cholesterol in a normal person following a 12-h fast? a. Very-low-density lipoproteins b. High-density lipoproteins c. Chylomicrons d. Chylomicron remnants e. Adipocyte lipid droplets

*The answer is C.* In mammals, arachidonic (5,8,11,15- eicosatetraenoic) acid can only be synthesized from essential fatty acids derived from the diet. Linoleic (9,12 octadecadienoic) acid produces arachidonic acid following two desaturations and chain elongation. While linolenic (9,12,15-octadecatrienoic) acid also is an essential fatty acid, desaturation and elongation produce 8,11,14,17-eicosatetraenoic acid, which is distinct from arachidonic acid. Oleic, palmitic, and stearic acids are all nonessential fatty acids that cannot give rise to arachidonic acids in mammals

Which of the following processes yields arachidonic (5,8,11,14- eicosatetraenoic) acid in mammals? a. Elongation of stearic acid b. Chain elongation and one desaturation of linolenic (9,12,15-octadecatrienoic) acid c. Chain elongation and two desaturations of linoleic (9,12 octadecadienoic) acid d. Desaturation of oleic acid e. Elongation of palmitic acid

*The answer is D.* Fatty acids must be activated before being oxidized. In this process, they are linked to CoA in a reaction catalyzed by thiokinase (also known as acyl CoA synthetase). ATP is hydrolyzed to AMP plus pyrophosphate in this reaction. In contrast, the enzyme thiolase cleaves off acetyl CoA units from β-ketoacyl CoA, while it forms thioesters during β oxidation.

Which of the following statements correctly describes the enzyme thiokinase? a. It yields acetyl CoA as a product b. It yields ADP as a product c. It yields CoA as a product d. It forms CoA thioesters as a product e. It requires β-ketoacyl CoA as a substrate

*The answer is D.* Regulation of cholesterol metabolism is by definition exerted at the "committed" and rate-controlling step. This is the reaction catalyzed by 3-hydroxy-3-methylglutaryl CoA reductase. Reductase activity is reduced by fasting and by cholesterol feeding and thus provides effective feedback control of cholesterol metabolism. The statin class of drugs act at this site.

Which of the following steps in the biosynthesis of cholesterol is thought to be rate-controlling and the locus of metabolic regulation? a. Geranyl pyrophosphate → farnesyl pyrophosphate b. Squalene → lanosterol c. Lanosterol → cholesterol d. 3-hydroxy-3-methylglutaryl CoA → mevalonic acid e. Mevalonic acid → geranyl pyrophosphate

*The answer is D.* The shell of apoproteins coating blood transport lipoproteins is important in the physiologic function of the lipoproteins. Some of the apoproteins contain signals that target the movement of the lipoproteins in and out of specific tissues. B48 and E seem to be important in targeting chylomicron remnants to be taken up by liver. B100 is synthesized as the coat protein of VLDLs and marks their end product, LDLs, for uptake by peripheral tissues. Other apoproteins are important for the solubilization and movement of lipids and cholesterol in and out of the particles. C-II is a lipoprotein lipase activator that VLDLs and chylomicrons receive from HDLs. The A apoproteins are found in HDLs and are involved in lecithin-cholesterol acyl transferase (LCAT) regulation.

Which one of the following apolipoproteins is synthesized in the liver as part of the coat of very-low-density lipoproteins (VLDLs)? a. AI b. B48 c. CII d. B100 e. E

*The answer is B.* Diacylglycerol-3-phosphate, more commonly known as phosphatidate, is an intermediate common to the synthesis of both triacylglycerol and phospholipids. In a two-step process, glycerol phosphate is successively acylated by two acyl CoAs to lysophosphatidate, which contains a fatty acid group in the 1′ position, and then phosphatidate, which contains fatty acid groups in the 1′ and 2′ positions with a phosphate group in the 3′ position. From that point, pathways for synthesis of phospholipids and triacylglycerol diverge. If storage lipid is to be produced, phosphatidate is dephosphorylated by a phosphatase and then acylated by acyl CoA to form triacylglycerol. In contrast, if phospholipids are to be produced, phosphatidate is activated by CTP in a reaction that produces CDP-diacylglycerol and pyrophosphate. Phosphatidylserine, phosphatidylinositol, phosphatidylethanolamine, and phosphatidylcholine can all be derived from CDP-diacylglycerol.

Which one of the following compounds is a key intermediate in the synthesis of both triacylglycerols and phospholipids? a. CDP-choline b. Phosphatidate c. Triacylglyceride d. Phosphatidylserine e. CDP-diacylglycerol

*The answer is B.* The TAG in chylomicrons are degraded to fatty acids and glycerol by lipoprotein lipase on the endothelial surface of capillaries in muscle and adipose, thus providing a source of fatty acids to these tissues for degradation or storage. In the duodenum, TAG are degraded to 1 monoacylglycerol + 2 free fatty acids that get absorbed. Medium and short chain fatty acids enter directly into blood; they do not get packaged into chylomicrons. Chylomicrons contain dietary lipids that were digested and absorbed, thus a defect in fat absorption would result in decreased production of chylomicrons.

Which one of the following statements about the absorption of lipids from the intestine is correct? A. Dietary triacylglycerol must be completely hydrolyzed to free fatty acids and glycerol before absorption. B. The TAG carried by chylomicrons is degraded to free fatty acids and glycerol by lipoprotein lipase on the endothelial surface of capillaries in muscle and adipose primarily. C. Fatty acids that contain ten carbons or less are absorbed and enter the circulation primarily via the lymphatic system. D. Deficiencies in the ability to absorb fat result in excessive amounts of chylomicrons in the blood.

*The answer is E.* Patients with CF, a genetic disease due to a deficiency of functional CFTR, have thickened secretions that impede the flow of pancreatic enzymes into the duodenum. The acid-stable lipases, lingual and gastric lipase, use as substrates TAG with short to medium-chain fatty acids that are abundant in milk. Emulsification occurs through peristalsis that provides mechanical mixing, and bile salts that function as detergents. Colipase restores activity to pancreatic lipase in the presence of inhibitory bile acids that bind the micelles. CCK (cholecystokinin) is the hormone that causes contraction of the gallbladder and release of stored bile; secretin causes release of bicarbonate. Chylomicron formation requires synthesis of the protein apolipoprotein B-48.

Which one of the following statements about the digestion of lipids is correct? A. Lipid digestion begins with acid-stable lipases that primarily use TAG with long to very long-chain fatty acids as substrates. B. Large lipid droplets are emulsified (have their surface area increased) in the mouth through the act of chewing (mastication). C. Colipase facilitates the binding of bile salts to mixed micelles, maximizing the activity of pancreatic lipase. D. The peptide hormone secretin causes the gallbladder to contract and release bile. E. Patients with cystic fibrosis have difficulties with digestion because their thickened pancreatic secretions are less able to reach the small intestine, the primary site of lipid digestion. F. Formation of chylomicrons is independent of protein synthesis in the intestinal mucosa.

*The answer is C.* Ceramide is the basic unit composing all sphingolipids, which include sphingomyelin and gangliosides. Sphingomyelin, which usually contains phosphocholine as a polar head group, is the only phospholipid that does not have a glycerol backbone. In contrast, gangliosides have complex oligosaccharide head groups.

Which one of the following steps results in the formation of a phospholipid? a. Step A b. Step B c. Step C d. Step D e. Step E

*The answer is A.* The evidence indicates that antiinflammatory steroids inhibit phospholipase A2, which is responsible for hydrolyzing arachidonate off of membrane phospholipids. Corticosteroids and their manufactured derivatives are thought to cause induction of the phospholipase A2-inhibitory protein lipocortin. In this manner, production of all of the derivatives of arachidonic acid (lipoxins, leukotrienes, thromboxanes, and prostaglandins) is shut off. In contrast, nonsteroidal anti-inflammatory agents such as aspirin, indomethacin, and ibuprofen act by inhibiting the cyclooxygenase component of prostaglandin synthase. The synthase is responsible for the first step in the production of prostaglandins (step E) and thromboxanes (step D) from arachidonic acid. The lipoxygenase pathway leads to the synthesis of lipoxins (step B) and leukotrienes (step C).

Which step in the diagram below is thought to be responsible for the effect of anti-inflammatory steroids? a. Step A b. Step B c. Step C d. Step D e. Step E

*The answer is D.* Because of the elevated liver enzymes (suggestive of liver damage), a statin would be relatively contraindicated in this patient, as a potential side effect of statins is liver damage. Cholestyramine would be a reasonable alternative to statins. Cholestyramine is one of the "bile acid binders" and prevents the reabsorption of bile salts. Since cholesterol is the precursor of bile salts, and 95% of bile salts are usually reabsorbed back into the enterohepatic circulation, losing bile salts in the feces would require increased synthesis of bile salts, thereby reducing the levels of free cholesterol in the body. Statins work by inhibiting HMG-CoA reductase. Cholestyramine does not reduce hepatic cholesterol synthesis, inhibit the release of bile salts, or interfere with the production of chylomicrons. Its sole action is in the lumen of the intestine, where it binds the bile salts so that they cannot be resorbed and sent back to the liver. The enterohepatic circulation is diagrammed below.

You have a patient whose blood work indicates high total cholesterol and elevated liver enzymes. You place him on cholestyramine to lower his cholesterol. Cholestyramine acts to lower cholesterol by inhibiting which of the following enzymes/pathways? (A) HMG-CoA reductase (B) Hepatic cholesterol synthesis (C) Release of bile salts from the gall bladder (D) Enterohepatic circulation reabsorption of bile salts (E) The production of chylomicrons

*The answer is C.* The first stage of cholesterol synthesis leads to the production of the intermediate mevalonate. Two molecules of acetyl-CoA condense to form acetoacetyl-CoA which condenses with another acetyl-CoA to form β-hydroxymethylglutaryl-CoA (HMG-CoA). HMGCoA synthase catalyses this step. Next, HMG-CoA reductase catalyzes the reduction of HMG-CoA to mevalonate. Statins (the class of drugs to which pravastatin belongs) directly inhibit HMG-CoA reductase, so mevalonate cannot be formed and cholesterol synthesis cannot continue. Statins do not inhibit the enzymes MCAD (required for fatty acid oxidation), CAT-1 (required for acyl-CoA transport into the mitochondria), or citrate lyase (needed to provide acetyl-CoA in the cytoplasm).

You have placed a patient on Pravachol pravastatin to reduce her cholesterol. This class of drugs is effective due to a direct inhibition of which of the following? (A) Medium chain acyl-CoA dehydrogenase (MCAD) (B) HMG-CoA synthase (C) HMG-CoA reductase (D) Carnitine acyltransferase 1 (CAT-1) (E) Citrate lyase

*The answer is C.* The patient has abetalipoproteinemia, an absence of apo B-containing proteins in the circulation. This leads to low chylomicron and VLDL levels. The problem is the synthesis of the chylomicrons and VLDL, both of which require the activity of the microsomal triglyceride transfer protein (MTTP). In the absence of MTTP activity, triglycerides cannot be transferred to the core particle as it is being synthesized, leading to little, if any, synthesis of these particles. The intestinal cells become laden with lipids obtained from the diet and those which cannot be exported due to the inability to produce chylomicrons. Mutations in LPL or apolipoprotein CII will not interfere with chylomicron or VLDL synthesis; mutations in those proteins would lead to an inability to remove triglyceride from those circulating particles. Deficiencies in LCAT or ABC1, which are related to HDL metabolism, would not affect the synthesis of chylomicrons or VLDL.

You see a patient who has steatorrhea, with very low levels of chylomicrons and VLDL in the circulation. Circulating triglyceride levels are extremely low. Examination of intestinal epithelial cells shows lipid-laden cells. A possible enzymatic defect leading to these findings is which of the following? (A) LPL (B) Apolipoprotein CII (C) MTTP (D) LCAT (E) CETP

*The answer is C.* The patient has dysbetalipoproteinemia, a mutation in apolipoprotein E, such that the patient exhibits the rare E2 form instead of the normal E3 form. Apolipoprotein E has affinity for the LDL receptor and the LDL receptor-related protein and, as such, is important for chylomicron remnant and IDL uptake from the circulation by the liver. With the homozygous E2 form, binding of the particles to their receptors is weak, and the particles circulate longer than normal, contributing to the high cholesterol and triglyceride levels seen in the circulation. Only about 10% of the individuals who are homozygous for E2 will develop this condition, and in those, obesity (BMI of 34) is a key factor which links the condition to the mutation. This disorder is not a problem with lipoprotein lipase (LPL) digesting triglycerides from particles, so neither LPL nor apo CII is defective. As both chylomicrons and VLDL are produced, it is not a defect in either apo B48 or B100 production or function.

Your 27-year-old male patient, with a BMI of 34, has a total cholesterol of 450 mg/dL and triglycerides of 610 mg /dL. He exhibits planar xanthomas and has already had one angioplasty last year. This patient may be exhibiting a rare autosomal recessive disorder which generates a mutation in which of the following proteins? (A) LPL (B) Apolipoprotein CII (C) Apolipoprotein E (D) Apolipoprotein B100 (E) Apolipoprotein B48

*The answer is B.* The patient is exhibiting the symptoms of diabetic ketoacidosis. Normal blood pH is in the range of 7.40. Diabetic ketoacidosis reduces the blood pH since ketone bodies accumulate and produce acid, which the blood has trouble buffering. A mild ketoacidosis would reduce the pH to about 7.25; one in which the patient exhibits neurological changes (weak and confused) would lower the pH even further to 7.15. Life-threatening diabetic ketoacidosis would be a pH of 7.0. Answers D and E are incorrect because they represent an alkalization of the blood, which does not occur when acids accumulate.

A 14-year-old girl with Type 1 diabetes has had viral gastroenteritis for 5 days, and she has been vomiting, been nauseous, and had trouble taking fluids by mouth. Because she was not eating, she did not take any insulin during her illness. She becomes weak and confused and is taken to the emergency room (ER) by her parents. The ER doctor notices a fruity odor to her breath, hyperventilation, and a blood glucose level of 600 mg/dL. A blood pH measurement is taken. You would expect this value to be which one of the following? (A) 6.75 (B) 7.15 (C) 7.40 (D) 7.65 (E) 8.00

*The answer is D.* The fruity odor is due to acetone, which is being exhaled. The acetone is derived from the spontaneous decarboxylation of acetoacetate (one of the ketone bodies) to acetone within the blood and tissues.

A 14-year-old girl with Type 1 diabetes has had viral gastroenteritis for 5 days, and she has been vomiting, been nauseous, and had trouble taking fluids by mouth. Because she was not eating, she did not take any insulin during her illness. She becomes weak and confused and is taken to the emergency room (ER) by her parents. The ER doctor notices a fruity odor to her breath, hyperventilation, and a blood glucose level of 600 mg/dL. The fruity odor noticed by the ER physician is due to which one of the following? (A) Oxidation of acetoacetate (B) Reduction of acetoacetate (C) Conversion of acetoacetate to acetoacetyl-CoA (D) Decarboxylation of acetoacetate (E) Carboxylation of acetoacetate

*The answer is A.* Hyperventilation is the body's way to try and raise the lowered blood pH by exhaling carbon dioxide rapidly. Carbon dioxide will form carbonic acid and a proton in the blood; as the carbon dioxide is exhaled, the acid and proton will associate so that the carbonic acid can form carbon dioxide and water. This will decrease the proton concentration in the blood, and raise the pH. The hyperventilation is not due to the altered hormonal ratios in theblood, the lack of fluids, or the lack of food.

A 14-year-old girl with Type 1 diabetes has had viral gastroenteritis for 5 days, and she has been vomiting, been nauseous, and had trouble taking fluids by mouth. Because she was not eating, she did not take any insulin during her illness. She becomes weak and confused and is taken to the emergency room (ER) by her parents. The ER doctor notices a fruity odor to her breath, hyperventilation, and a blood glucose level of 600 mg/dL. The patient is hyperventilating because of which one of the following? (A) The low pH of the blood (B) The elevated pH of the blood (C) The increased glucagon/insulin ratio in the blood (D) Lack of fluids in the body (E) Difficulty in breathing due to the lack of food

*The answer is B.* The baby has respiratory distress syndrome, due to an inability to produce surfactant, a hydrophobic molecule that is secreted by the type II cells in the lung and coats the airways, reducing surface tension during contraction, and allowing relatively easy expansion of the lung during inhalation. This is due to the lungs not yet producing surfactant, which contains a few proteins and a large amount of dipalmitoylphosphatidyl choline. Respiratory distress syndrome is not related to insulin or glucagon response by the lung, or the ability of the lung cells to generate energy.

A 2-day-old infant born at 32 weeks gestation has had breathing difficulties since birth and is currently on a respirator and 100% oxygen. These difficulties occur due to which one of the following? (A) An inability of the lung to contract to exhale (B) An inability of the lung to expand when taking in air (C) An inability of the lung to respond to insulin (D) An inability of the lung to respond to glucagon (E) An inability of the lung to produce energy

*The answer is C.* This infant most likely has medium-chain acyl CoA dehydrogenase (MCAD) deficiency. MCAD is important in the beta-oxidation of fatty acids after the initial cycles of this process are carried out by long-chain aryl CoA dehydrogenase (LCAD) to yield fragments 8 to 10 carbons in length. Without the MCAD enzyme, these medium chain length fatty acids accumulate, eventually spilling out of the mitochondria via the carnitine shuttle. The resultant C8-C10 acylcarnitines are sufficiently amphipathic that they can exit the cells and spill into the bloodstream. Levels of C8-C10 acylcarnitines in the blood are measured by mass spectrometry to diagnose MCAD deficiency. The inability to conduct fatty acid beta-oxidation (which requires the MCAD enzyme) in the liver during fasting conditions results in low ATP levels. The urea cycle is operating maximally in the liver during fasting conditions, but energy is required, and with low ATP levels, the activity of the urea cycle decreases, and ammonia builds up in the bloodstream. In the early neonatal period, the newborn is often asymptomatic because frequent feedings protect the newborn from the need to cope with the metabolic stresses of fasting. The family is typically initially unaware of the problem, especially as this is an autosomal recessive condition, so close relatives may be unaffected. As the infant gets older and feeding frequency decreases, symptoms that may develop (particularly at night) are often related to hypoglycemia, such as irritability before meals, lethargy, jitteriness, sweating, and, possibly, seizures. By about 3 months of age, the infant may become seriously ill after a viral illness, with symptoms including lethargy accompanied by vomiting and acidosis. Early recognition of the problem is important because an affected infant may die during a severe episode. Management is based on a diet designed to ensure feedings at no more than 5-hour intervals.

A 2-month-old boy with mildly dysmorphic features is brought to the emergency department after having a brief generalized seizure, which left him semicomatose. Laboratory studies shew elevated C8-C10 acylcarnitines. Which of the following additional findings is most likely in this patient? A. Decreased secretion of glucagon B. Decreased plasma free fatty acids C. Hyperammonemia D. Hyperglycemia E. Hyperketosis

*The answer is A.* The child has Zellweger's syndrome, an absence of peroxisomal enzyme activity. Of the pathways listed as answers, only the oxidation of very long chain fatty acids is a peroxisomal function. Fatty acid synthesis occurs in the cytoplasm. Acetyl-CoA oxidation takes place in the mitochondria. Glucose oxidation is a combination of glycolysis (cytoplasm) and the TCA cycle (mitochondria). Triglyceride synthesis occurs in the cytoplasm.

A 2-month-old infant with failure to thrive displays hepatomegaly, high levels of iron and copper in the blood, and vision problems. This child has difficulty in carrying out which of the following types of reactions? (A) Oxidation of very long chain fatty acids (B) Synthesis of unsaturated fatty acids (C) Oxidation of acetyl-CoA (D) Oxidation of glucose (E) Synthesis of triacylgycerol

*The answer is C.* The combination of xanthomas in the Achilles tendon, arcus lipoides (the opaque rings found on the edge of the iris), and very high LDL level is pathognomonic for familial hypercholesterolemia, or type IIa familial dyslipidemia. This autosomal dominant disease is due to defects in the LDL receptor~ which is responsible for removing LDL from the circulation in the liver. Heterozygous patients have elevated levels of LDL, which may manifest in middle age; homozygous patients, however, may suffer a myocardial infarction in the first decade of life.

A 2-year-old girl has xanthomas on her Achilles tendon and opaque rings on the lower edge of her iris. Laboratory tests show normal triglyceride and HDL levels, but an LDL level of 750 mg/dL A primary defect or deficiency in which of the following is most likely in this patient? A. Apolipoprotein C-II B. Apolipoprotein E C. LDL receptors D. Lipoprotein lipase E. VLDL production

*The answer is C.* The symptoms in this patient are consistent with vitamin A deficiency, which is characterized by night blindness, dry conjunctivae, gray plaques, and later development of corneal ulceration and necrosis leading to perforation and blindness. This deficiency is typically seen in children and pregnant women whose diets are deficient in vitamin A. It can also be seen in alcoholics and in patients with fat malabsorption, cholestasis, or inflammatory bowel disease. Vitamins A, D, E, and K (fat-soluble vitamins) are absorbed in the small intestine and absorption requires micelles formed with bile salts. Vitamin A is absorbed in the duodenum.

A 2-year-old girl who has recently been adopted from Southeast Asia is brought to the clinic by her adopted parents. They are concerned because the child seems to be having trouble with her vision in low-light conditions. The nutrient most likely deficient in this child is absorbed by the gastrointestinal system using what mechanism? A. Apoferritin-mediated transport B. Intrinsic factor-mediated transport C. Micelle-mediated transport D. Sodium-dependent cotransport E. Vitamin D-dependent binding protein-mediated transport

*The answer is A.* Hypertriglyceridemia (triglyceride [TG] > 1000 mg/dL) may cause acute pancreatitis. Therefore, in a patient with a history of pancreatitis, pharmacologic treatment for a serum TG level >500 mg/dL is recommended as primary therapy to prevent recurrence. (A normal TG is considered to be < 150 mg/dl.) Fibrates decrease the levels of VLDL cholesterol and slightly reduce the levels of LDL. Fibrates activate the nuclear transcription factor, peroxisome proliferator-activated receptor-a (PPARa), which increases lipoprotein TG lysis via lipoprotein lipase and increases HDL levels.

A 22-year-old man visits his primary care physician after recently being discharged from the hospital following treatment for pancreatitis. He denies smoking, alcohol use, or illicit drug use. He reports that his maternal grandfather died from colon cancer at the age of 76. Other than the recent hospitalization for pancreatitis, his medical history is significant only for gout. Currently, the patient has a serum triglyceride level of more than 1000 mg/dL. What is the mechanism of the first-line pharmaceutical intervention indicated for this patient? A. Increases activity of lipoprotein lipase B. Inhibition of 3-hydroxy-3-methylglutaryl coenzyme A reductase C. Inhibits intestinal absorption of cholesterol D. Promotes lipolysis by activat ing hormone sensit ive-lipase E. Sequestration of charged bile acids

*The answer is D.* This patient most likely has familial hypercholesterolemia, an autosomal dominant disease with a heterozygote frequency of 1 in 500. This disorder is characterized by abnormal LDL receptors. Seventy-five percent of LDL receptors are found within the liver. This receptor binds to apolipoprotein B and apolipoprotein E, which are found on intermediate-density lipoprotein (IDL) and LDL particles. The inability of LDL to bind these proteins results in a higher serum concentration of LDL and also a higher rate of conversion of IDL into LDL. Elevated serum LDL levels are found on laboratory tests. Deposition of LDL also occurs widely. Skin deposition results in xanthomas, and vascular deposition results in atherosclerosis.

A 25-year-old man presents to his primary care physician for a physical examination before starting a new job. His family history is significant for a father and grandmother with "cholesterol issues." For this reason the patient has maintained a healthy diet and exercises 5 days per week. Physical examination reveals a well-developed, well-nourished adult man with the xanthelasma. Laboratory studies show a total cholesterol level of 300 mg/dl. A mutation in which protein most likely accounts for this patient's dyslipidemia? A. Apolipoprotein A-II B. Apolipoprotein C-II C. Apolipoprotein E D. LDL receptor E. Lipoprotein lipase deficiency F. PKD1

*The answer is B.* Dietary modification (drastically limiting saturated and trans fats and cholesterol), weight loss, and aerobic exercise are the first-line treatment options for any patient with elevated cholesterol levels. Although these lifestyle modifications should be attempted by this patient, they often have only minimal effect. Statin medications are 3-hydroxy-3-methylglutaryl coenzyme A (HMG-CoA) reductase inhibitors. By blocking the rate-limiting step in cholesterol synthesis, they can increase hepatic synthesis of LDL receptors, thereby lowering serum LDL levels. The toxicity of statin medicat ions includes myopathy, which is causing this patient's symptoms, and elevated creatine kinase levels. Rarely, patients can develop rhabdomyolysis with renal failure. Elevated liver enzyme levels also can be seen with statin treatment, although this usually is reversible.

A 25-year-old woman presents to her family physician for a routine check-up. Physical examination reveals a mildly overweight woman, but is otherwise unremarkable. A fasting lipid panel shows an LDL cholesterol level of 310 mg/dl, HDL cholesterol level of 42 mg/dl, triglyceride level of 150 mg/dl, and total cholesterol level of 382 mg/dl. The doctor initiates t reatment of her condit ion. Soon after starting treatment, she presents with myalgias. Laboratory tests show elevated levels of aspartate aminotransferase, alanine aminotransferase, and serum creatine kinase. Which of the following describes the mechanism of the intervention that was most likely initiated to treat this patient 's hypercholesterolemia? A. Activates peroxisome proliferator-activated receptor alpha B. Inhibition of 3-hydroxy-3-methylglutaryl coenzyme A reductase C. Inhibition of bile acid reuptake in the intestine D. Inhibition of cholesterol uptake by the intestinal brush border E. Reduced transfer of cholesterol from HDL to LDL and delayed HDL clearance

*The answer is D.* The patient most likely has familial hypercholesterolemia due to a mutated LDL receptor. Lovastatin is an inhibitor of HMG-CoA reductase. HMG-CoA reductase inhibitors cause cells to decrease the rate of cholesterol synthesis. Lower cellular levels of cholesterol cause a decreased conversion of cholesterol to cholesterol esters (by the ACAT reaction) for storage and an increased production of LDL receptors. An increased number of receptors will cause more LDL to be taken up by cells from the circulation and degraded by lysosomes. Thus, blood cholesterol levels will decrease. Blood triacylglycerol levels will also decrease but not to a great extent because LDL contains only small amounts of triacylglycerol. The patient most likely is heterozygous for a mutation in the LDL receptor. If the patient were homozygous for such a mutation, then lovastatin would not reduce circulating cholesterol levels, as there would be no functional LDL receptors to upregulate. The patient does not have a mutation in HMG-CoA reductase, as that is the target of lovastatin, and if an HMGCoA reductase mutation were to reduce HMG-CoA reductase activity, then lovastatin would have little to no effect. A mutation in protein kinase A, rendering it inactive, would not affect intracellular cholesterol synthesis (as it is the AMP-activated protein kinase that regulates the HMG-CoA reductase activity). An inactivating mutation in ACAT would not lead to a reduction of circulating cholesterol in response to lovastatin. Inactivating mutations in lipoprotein lipase lead to hypertriglyceridemia, and do not respond to lovastatin.

A 28-year-old man was found to have elevated cholesterol levels of 325 mg/dL on a routine checkup. His father died of a heart attack at the age of 42, and also had greatly elevated cholesterol levels throughout his life. The man's physician placed him on lovastatin, and his cholesterol levels dropped to 170 mg/dL. The nature of the elevated cholesterol in this patient is most likely due to a mutation in which one of the following proteins? (A) HMG-CoA reductase (B) Protein kinase A (C) ACAT (D) LDL receptor (E) Lipoprotein lipase

*The answer is C.* Defects in fatty acid oxidation deprive the liver of energy when fatty acids are the major energy source (such as during exercise, or a fast). Because of this, there is insufficient energy to synthesize glucose from gluconeogenic precursors (it requires 6 moles of ATP to convert 2 moles of pyruvate to 1 mole of glucose). Acyl-carnitines and dicarboxylic acids have no effect on the enzymes of gluconeogenesis, nor do they hinder the ability of the red blood cell to utilize glucose through the glycolytic pathway. Additionally, acetyl-CoA levels are low due to the lack of complete fatty acid oxidation and pyruvate carboxylase, a key gluconeogenic enzyme, is not fully activated. This also contributes to the reduced gluconeogenesis observed in patients with MCAD defective.

A 3-month-old child had her first ear infection and was feeding poorly due to the ear pain. One morning the parents found the child in a nonresponsive state and rushed her to the emergency department. A blood glucose level was 45 mg/dL, and upon receiving intravenous glucose the child became responsive. Further blood analysis displayed the absence of ketone bodies, normal levels of acylcarnitine, and the presence of the following unusual carboxylic acids shown below. Regarding this child, why were fasting blood glucose levels so low? (A) Acyl-carnitine inhibition of gluconeogenesis (B) Dicarboxylic acid inhibition of gluconeogenesis (C) Insufficient energy for gluconeogenesis (D) Dicarboxylic acid inhibition of glycogen phosphorylase (E) Reduction of red blood cell production of lactate for gluconeogenesis

*The answer is E.* The principal route of metabolism of ethanol is via alcohol dehydrogenase and acetaldhyde dehydrogenase, both of which reduce NAD+ to NADH, markedly increasing the ratio of NADH to NAD+. The relative excess of NADH has a number of effects, including inhibiting, rather than stimulating, fatty oxidation, inhibiting gluconeogenesis rather than stimulating it. The higher than normal NAD/NAD+ favors the formation of glycerol 3-phosphate from dihydroxyacetone phosphate and the formation of lactate from pyruvate.

A 45-year-old man drinks 5-6 alcoholic drinks per day. Which of the following metabolic alterations is most likely to be present in this man, compared to a nondrinker? A. Dihydroxyacetone phosphate formation is favored over glycerol-3-phosphate formation B. Fatty acid oxidation is stimulated C. Gluconeogenesis is stimulated D. The ratio of lactate to pyruvate is decreased E. The ratio of NADH to NAD + is increased

*The answer is E.* The child has MCAD (medium-chain acyl-CoA dehydrogenase) deficiency, an inability to completely oxidize fatty acids to carbon dioxide and water. With an MCAD deficiency, gluconeogenesis is impaired due to a lack of energy from fatty acid oxidation, and an inability to fully activate pyruvate carboxylase, as acetyl-CoA activates pyruvate carboxylase, and acetyl-CoA production from fatty acid oxidation is greatly reduced. In an attempt to generate more energy, medium-chain fatty acids are oxidized at the ω ends to generate the dicarboxylic acids seen in the question. The finding of such metabolites (dicarboxylic acids) in the blood is diagnostic for MCAD deficiency. If there were mutations in any aspect of carnitine metabolism, there would be no oxidation of fatty acids (the fatty acids would not be able to enter the mitochondria), and the dicarboxylic acids (which are byproducts of fatty acid metabolism) would not be observed. Similarly, a mutation in the fatty acyl-CoA synthetase (the activating enzyme, converting a free fatty acid to an acyl-CoA) would also result in a lack of fatty acid oxidation, as fatty acids are not able to enter the mitochondria in their free (nonactivated) form.

A 3-month-old child had her first ear infection and was feeding poorly due to the ear pain. One morning the parents found the child in a nonresponsive state and rushed her to the emergency department. A blood glucose level was 45 mg/dL, and upon receiving intravenous glucose the child became responsive. Further blood analysis displayed the absence of ketone bodies, normal levels of acylcarnitine, and the presence of the following unusual carboxylic acids shown below. The enzymatic defect in this child is most likely in which of the following enzymes? (A) Fatty acyl-CoA synthetase (B) Carnitine translocase (C) Carnitine acyltransferase I (D) Carnitine acyltransferase II (E) Medium chain acyl-CoA dehydrogenase

*The answer is A.* This patient has myopathy (og, elevated creatine kinase, weakness), cardiomyopathy (eg, S3 gallop) and hypoketotic hypoglycemia in the setting of decreased muscle carnitine content, which is consistent with primary carnitine deficiency. This condition is caused by a defect in the protein responsible for carnitine transport across the plasma membrane. Without sufficient carnitine, fatty acids cannot be transported from the cytoplasm into the mitochondria as acyl-camitine (carnitine shuttle). The mitochondria therefore cannot (β-oxidize the fatty acids into acetyl CoA,the carbon substrate for the citric acid cycle. As a result, cardiac and skeletal myocytes cannot generate ATP from fatty acids and the liver is unable to synthesize ketone bodies (je, acetoacetate) when glucose levels are low. Hypoketotic hypoglycemia is also seen in other fatty acid oxidation disorders (ag, rriadium- or very-long-chain acyl CoA dehydrogenase deficiency).

A 3-year-old girl is brought to the clinic due to several months of fatigue and difficulty walking. She ambulates normally at first but rapidly becomes weak and tired. The patient has not been ill recently and is usually happy and playful. She has a history of mild motor delays but is otherwise developmentally normal. Vital signs are within normal limits. Examination shows mildly decreased power in all extremities but no ataxia. Cardiac auscultation reveals 1/6 systolic murmur and an S3 gallop. Laboratory results are as follows: Serum chemistry Glucose 37 mgldL Creatine kinase 304 U/L Urinalysis Protein none Glucose negative Ketenes negative Leukocyte esterase negative Nitrites negative Muscle biopsy shows a very low carnitine content. Which of the following substances has deficient synthesis in this patients disease? (A) Acetoacetate (B) Arachidonic acid (C) Glutathione (D) Homocysteirme (E) Lactate (F) Palmitate

*The answer is A.* During a period of prolonged starvation. the following biochemical events take place 1. Triglyceride in adipose is hydrolyzed. releasing fatty acids for tissues that can use them including liver, adipose, and muscle, In these tissues. fatty acids are metabolized by β-oxidation, producing acetyl-CoA 2. In liver, extra acetyl-CoA will accumulate and be used for ketone body (primarily acetoacetate and 3-hydro1cybutyrate) synthesis 3. Ketones will be metabolized in muscle and. after a week of starvation. in the brain. This spares glucose for tissues relying on anaerobic glycolysis (RBC) and also, importantly slows the breakdown of essential body protein. Free glycerol is also a product of triglyceride hydrolysis. It can be phosphorylated to glycerol-3-phosphate in liver and used for gluconeogenesis. Glycerol-3-phosphate makes up a larger percentage of substrate for gluconeogenesis in ooese individuals

A 30-year-bid woman is brought to the emergency department by a friend because of excessive weight loss, The patient's friend says that the patient has lost 27.2 Kg (60 lb) over the past 4 months by consuming just water and vitamin pills. Which of the following will most likely be elevated on laboratory studies? A. Acetoacetic acid B. Alanine C. Bicarbonate D. Chylomicrons E. VLDL

*The answer is C.* Type I dyslipidemia (or familial lipoprotein lipase deficiency) is caused by a deficiency of lipoprotein lipase. This enzyme exists in capillary walls of adipose and muscle tissue and cleaves triglycerides into free fatty acids and glycerol. The enzyme is activated by apolipoprotein C-II, which is found on VLDL cholesterol and chylomicrons. Type I dyslipidemia is characterized by an accumulation of triglyceride-rich lipoproteins in the plasma. Deficiency in apolipoprotein C-II produces a similar result.

A 30-year-old man is diagnosed with type I familial dyslipidemia. Recent laboratory studies show an elevated triglyceride level but normal LDL and HDL cholesterol levels. Which of the following explains the pathophysiology of this disease? (A) Apolipoprotein E deficiency (B) LDL cholesterol receptor deficiency (C) Lipoprotein lipase deficiency (D) VLDL cholesterol clearance deficiency (E) VLDL cholesterol overproduction

*The answer is C.* This young man most likely has type I dyslipidemia, or familial hyperchylomicronemia. This condition is caused by a deficiency of lipoprotein lipase, an enzyme found in capillary walls of adipose and muscle tissue. Its purpose is to digest chylomicrons by cleaving triglycerides into free fatty acids and glycerol that can then be used by nearby tissues. Deficiency of lipoprotein lipase results in inability to effectively utilize chylomicrons, lead ing to excessively high levels of chylomicrons and triglycerides in the blood. This excess in fatty particles can sometimes be seen by the naked eye as a milky-white discoloration to the blood. The resultant high levels of serum triglycerides (often >1000 mg/dL) can greatly increase the risk of pancreatitis in these patients. Interestingly, cardiac risk in these patients is not increased because of usually normal levels of LDL and HDL.

A 30-year-old previously healthy man is seen in clinic for follow-up several weeks after a bout of pancreatitis. The patient does not drink alcohol and has no history of gallstones. He is neither obese nor diabetic. In clinic, blood is drawn and is milky white in color. Which of the following most likely explains the pathophysiology of his underlying condition? A. Defect in processing LDL receptors B. Increase in apolipoprotein CII levels C. Lipoprotein lipase deficiency D. Mutat ion in apolipoprotein E E. VLDL cholesterol clearance deficiency

*The answer is B.* Familial dysbetalipoproteinemia (type Ill hyperlipoproteinemia) is characterized clinically by xanthomas and premature coronary and peripheral vascular disease. The primary defects in familial dysbetalipoproteinemia are in ApoE3 and ApoE4, apolipoproteins found on chylomicrons and VLDL that are responsible for binding hepatic apolipoprotein receptors. Without ApoE3 and ApoE4, the liver cannot efficiently remove chylomicrons and VLDL remnants from the circulation, causing thechir accumulation in the serum and resultant elevations in cholesterol and triglyceride levels. *(Choice A)* Chylomicrons, particles composed primarily of triacylglycerol, are synthesized on the RER and Golgi apparatus of small intestinal enterocytes. They are released from enterocytes with only ApoB-48 apolipoprotein, they subsequently receive ApoC-Il and ApoE from HDL particles. *(Choice C)* ApoB-100 is present on LDL and is required for receptor-mediated uptake of LDL by extrahepatic tissues. *(Choice D)* Lipoprotein lipase is activated by ApoC-ll carried by chylomicrons and VLDL. ApoC-ll deficiency results in hyperchylomicronemia (type 1 hyperlipoproteinemia) *(Choice E)* ApoA-l is required for esterification of free cholesterol in HDL particles by lecithin-cholesterol acyltransferase (LCAT). ApoA-I and LCAT deficiencies result in low HDL end increased circulating free cholesterol levels. *Educational objective:* The key functions of important apolipoproteins are as follows: *ApoA-I:* LCAT activation (cholesterol esterification) *ApoB-48:* Chylomicron assembly and secretion by the intestine *ApoB-100:* LDL particle uptake by extrahepatic cells *ApoC-ll:* Lipoprotein lipase activation *ApoE-3 & -4:* VLDL and chylomicron remnant uptake by liver cells

A 36-year-old male has orange-yellow discoloration of his palmer creases and small clusters of yellow papules on his elbows, knees and buttocks. Laboratory evaluation suggests a lack of ApoE3 and ApoE4 in his circulating lipoproteins. Which of the following is most likely impaired in this patient? (A) Chylomicron secretion by the intestine (B) Chylomicron remnant uptake by liver cells (C) LDL particle uptake by extrahepatic cells (D) Lipoprotein lipase activation (E) Cholesterol esterificatlon in the blood

*The answer is C.* This patient's signs and symptoms in the setting of venous stasis (a long airplane flight) are most consistent with a deep vein thrombosis (DVT). Clinical suspicion of a DVT is increased with a positive Homan sign (calf pain on dorsiflexion), which is seen in some patients with a DVT. Patients with Virchow triad (local trauma to the vessel wall, stasis, and hypercoagulability) are at increased risk for developing DVT. Thromboxane A2, produced by platelets, stimulates platelet aggregation and vasoconstriction and will be elevated at the site of a clot. Thromboxane A2 also acts as an amplification signal for more potent platelet agonists, such as thrombin.

A 36-year-old woman presents to her primary care physician complaining of left calf pain. The pain began yesterday and worsens with walking. She denies any history of trauma to the extremity, but does report that she returned from a business trip to Japan 4 days ago. On physical examination, the patient is afebrile with a heart rate of 82/min, blood pressure of 129/86 mm Hg, respiratory rate of 14/min, and oxygen saturation of 98%. The left lower extremity is larger in diameter than the right. Which of the following best describes the series of events resulting in this patient's condition? A. Elevation of prostaglandin I2 → to platelet aggregation and vasoconstriction B. Elevation of thromboxane A2 → to increased platelet production C. Increased thromboxane A2 → to platelet aggregation and vasoconstriction D. Inhibition of prostaglandin I2 → to increased platelets production E. Release of leukotriene B4 from vascular endothelium

*The answer is B.* The child has the symptoms of primary carnitine deficiency. Carnitine cannot be transported from the blood into the liver and muscle, and fatty acid oxidation in those tissues is severely impaired. The inability to utilize fatty acids for energy give rise to muscle weakness, and an accumulation of fatty acids can occur within the muscle tissue. The inability of the liver to oxidize fatty acids will lead to fasting hypoglycemia as there is insufficient energy for gluconeogenesis. An MCAD deficiency would not show fatty infiltration in the muscle, nor elevated levels of free fatty acids (the presence of medium-chain dicarboxylic acids, or acylglycines, would be observed instead). A defect in carnitine acyltransferase II would result in acylcarnitines in the circulation. An HMG-CoA synthase deficiency would not allow ketone body formation, and would not present with these symptoms. A lack of acetyl-CoA carboxylase would greatly reduce the fatty acid content within the fat cell, as endogenous fatty acids would not be able to be synthesized from acetyl-CoA.

A 4-month-old child exhibited extreme tiredness, irritable moods, poor appetite, and fasting hypoglycemia associated with vomiting and muscle weakness. Blood work showed elevated levels of free fatty acids, but low levels of acylcarnitine. A muscle biopsy demonstrated a significant level of fatty acid infiltration in the cytoplasm. The most likely molecular defect in this child is in which one of the following enzymes? (A) Medium-chain acyl-CoA dehydrogenase (B) Carnitine transporter (C) Acetyl-CoA carboxylase (D) Carnitine acyltransferase II (E) HMG-CoA synthase

*The answer is C.* A deficiency in acetyl-CoA carboxylase can lead to a dysfunction in fatty acid synthesis and may lead to growth delay, myopathy, and muscular hypotonia. Fatty acid synthesis takes place using malollyl-coenzyme A (CoA), a 3-carbon unit, as the building block. Malonyl-CoA is formed by carboxylating acetyl-CoA via the enzyme acetyl-CoA carboxylase, using biotin as a cofactor. One molecule of acetyl-CoA is joined with one molecule of carbon dioxide aided by ATP. Malonyl-CoA is utilized in fatty acid biosynthesis by the enzyme MCAT (malonyl CoA:acyl carrier protein transacylase), which transfers malonate from malonyl-CoA to the terminal thioester on the acyl carrier protein. Malonyl-CoA is an important regulatory factor in the fatty acid synthesis/degradation pathway. It inhibits the rate-limiting step in β-oxidation of fatty acids (association of fatty acids with carnitine), thereby preventing them from entering the mitochondria where fatty acid degradation occurs .

A 4-year-old boy is brought to the physician because he is not growing as quickly as the neighborhood children. The mother is extremely worried about the developmental delay and states that he has been behind on all his motor skill landmarks growing up. Upon examination, the physician notices the boy is in the fifth percentile for weight and height for his age and has signs of muscle hypotonia. The physician is determined to find out the cause for the patient's developmental delay and sends him to a specialist. It is later determined that the patient has a deficiency in fatty acid synthesis, specifica lly a deficiency in acetyl-CoA carboxylase and is not able to produce the essential building block for fatty acid synthesis. Which of the following is the building block for fatty acid synthesis? A. Glycerol-3-phosphate B. Ketone bodies C. Malonyl-coenzyme A D. Reduced nicotinamide adenine dinucleotide phosphate E. 5-Adenosylmethionine

*The answer is A.* Grapefruit juice contains furanocoumarins, which inhibit the cytochrome P450 enzyme CYP3A4 that prepares statins for excretion. Grapefruit juice does not act as an inhibitor of HMG-CoA reductase. Lovastatin binds to HMG-CoA reductase, but not to cholesterol.

A 50-year-old male patient has high cholesterol levels and is placed on lovastatin. He is counseled to stop drinking his usual glass of grapefruit juice every morning. Given that grapefruit juice interferes with lovastatin action, which one of the following best explains this interaction? (A) Grapefruit juice inhibits the cytochrome p450 enzyme which modifies lovastatin for excretion. (B) Grapefruit juice stimulates the cytochrome p450 enzyme which modifies lovastatin for excretion. (C) Grapefruit juice is a competitive inhibitor of lovastatin binding to cholesterol. (D) Grapefruit juice is a competitive inhibitor of HMG-CoA reductase. (E) Grapefruit juice reduces the maximal velocity of HMG-CoA reductase.

*The answer is B.* Grapefruit juice contains furanocoumarins, which inhibit the cytochrome P450 complex CYP3A4. This complex modifies various statins for rapid excretion from the body. Thus, in the presence of grapefruit juice, statin levels will be higher than expected. This will lead to prolonged inhibition of HMG-CoA reductase and a reduction of cholesterol levels (with minimal effect on HDL levels), but will also increase the probability of side effects from statin treatment, one of which is muscle pain and weakness. The grapefruit juice plus statin will not lead to gastric reflux or steatorrhea.

A 50-year-old male patient has high cholesterol levels and is placed on lovastatin. He is counseled to stop drinking his usual glass of grapefruit juice every morning. Which of the following may occur when someone taking lovastatin chronically consumes grapefruit juice? (A) Cholesterol levels increase (B) Muscle pain and discomfort (C) Steatorrhea (D) Acid reflux (E) A decrease in HDL levels

*The answer is C.* Platelet aggregation is often a determining factor in heart attacks. Thromboxane A2, produced by platelets, promotes platelet aggregation when clotting is required, and inhibition of thromboxane A2 synthesis by aspirin reduces the potential for inappropriate clot formation, and further heart attacks. Thromboxane A2 is produced from arachidonic acid by the action of cyclooxygenase, the enzyme covalently modified and irreversibly inhibited by aspirin. Leukotrienes are also synthesized from arachidonic acid, but utilize lipoxygenase in their synthesis, which is not inhibited by aspirin. Cholesterol, triglyceride, and cytokine synthesis do not require cyclooxygenase activity.

A 52-year-old man, after suffering a heart attack, was put on 81 mg of aspirin daily by his cardiologist. The purpose of this treatment is to reduce the levels of which one of the following? (A) Cytokines (B) Leukotrienes (C) Thromboxanes (D) Cholesterol (E) Triglycerides

*The answer is B.* This patient is suffering from hypoglycemia related to his alcohol use. Ethanol is metabolized to acetaldehyde, which is then metabolized to acetate (acetaldehyde dehydrogenase). During both steps of ethanol metabolism, NADH is generated from NAD+. With an elevated NADH: NAD+ ratio in the liveer, pyruvate is diverted to lactate, while oxaloacetate is diverted to malate, thus inhibiting gluconeogenesis. The metabolism of ethanol is also responsible for the hepatic fatty changes seen in chronic alcoholics (increased fatty acid synthesis).

A 54-year-old alcoholic is brought to the emergency department by fire rescue after being found lying face down in the street. He is incoherent and is unable to walk in a straight line. His pulse is 110/min, his blood pressure is 135/80 mm Hg, and his respiration rate is 20/min. Physical examination reveals a diaphoretic man with generalized weakness passing in and out of consciousness. A glucose fingerstick test shows a glucose level of 45 mg/ dl. This patient's hypoglycemia most likely resulted from an elevated ratio of which of the following? A. NAD+: NADH B. NADH: NAD+ C. NADP+: NADPH D. NADPH: NADP+ E. Pyruvate: lactate

*The answer is C.* Thromboxane A2 release from platelets is an essential element of forming blood clots, and aspirin will block prostaglandin, prostacyclin, and thromboxane synthesis. It is the thromboxane inhibition which reduces the risk of blood clots. Leukotrienes and lipoxins require the enzyme lipoxygenase, which is not inhibited by aspirin. These pathways are outlined below.

A 55-year-old man had been advised by his physician to take 81 mg of aspirin per day to reduce the risk of blood clots leading to a heart attack. The rationale for this treatment is which of the following? (A) To reduce prostaglandin synthesis (B) To reduce leukotriene synthesis (C) To reduce thromboxane synthesis (D) To increase prostacyclin synthesis (E) To increase Lipoxin synthesis

*The answer is D.* Ethanol is broken down to acetaldehyde by alcohol dehydrogenase, and then to acetate by acetaldehyde dehydrogenase. Both steps occur in the liver and require reduction of NAD+ to its NADH counterpart, thus increasing the overall cell NADH: NAD+ ratio. In chronic alcoholics, the overall increased NADH: NAD+ ratio in hepatocytes leads to changes in metabolic pathways, such as inhibition of fatty acid oxidation. This helps promote fatty changes in the liver. Prolonged fat ty changes and repeated damage to the liver leads to scarring and cirrhosis of the liver.

A 55-year-old man is brought to t he emergency department by police after sustaining a laceration in a street fight outside of a bar. He was detained by the police for several hours before coming to the hospital, and on examination, he is diaphoretic and tremulous. His liver is firm 12 cm below the right costal margin. What is the biochemical process that explains the chronic changes to this man's liver? A. Atherosclerosis in the liver B. Decreased NADH: NAD+ ratio C. Increase in the citric acid cycle D. Increased NADH: NAD+ ratio E. Increased protein synthesis

*The answer is B.* This patient was started on an 3-hydroxy-3 methylglutaryl coenzyme A (HMG CoA) reductase inhibitor (statin), which prevents the conversion of HMG CoA to mevalonic acid, the rate-limiting step in cholesterol biogenesis. HMG CoA is comformed from three acetyl CoA molecules and is a precursor to sterols and ketone bodies. Muscle pain or injury resembling myositis has been known to occur with statins. Although the mechanism is unknown, it may be related to a decrease in muscle tissue synthesis of ubiquinone, a coenzyme used in muscle cell metabolism.

A 57-year-old woman visits her primary care physician. Laboratory studies reveal an LDL of 194 mg/dL and HDL of 41 mg/dL. Her physician begins therapy with a drug that inhibits production of mevalonic acid. Which of the following is a common side effect of this therapy? (A) Hepatomegaly without elevations in aspartate aminotransferase or alanine aminotransferase (B) Muscle injury clinically similar to myositis (C) Spontaneous abortion of a pregnancy (D) Suicidality and homicidality in patients with bipolar disorder (E) Tonic-clonic seizures

*The answer is E. Following endothelial injury, the subendothelial space accumulates lipoproteins. Next, chemical modification (eg, glycation or oxidation) of lipoproteins occurs that recruits monocytes to the vessel wall. Monocytes are converted to macrophages upon entry into the subendothelial space, and unregulated macrocytosis of modified LDL cholesterol occurs, yielding foam cells. At this point, various cell mediators, most notably platelet-derived growth factor, tumor necrosis factor, and interleukin-1, recruit platelets and smooth muscle to the intimal lining, where proliferation and production of extracellular matrix leads to the development of a fibrous plaque.

A 59-year-old woman with history of morbid obesity, hypercholesterolemia, and diabetes mellitus presents to the emergency department with complaints of substernal chest pain lasting two hours. An ECG reveals ST elevations in the lateral leads. The troponin level at admission is extremely elevated, and a creatine kinase-myocardial bound test is pending. Which of the following is a key cell mediator in the pathogenesis of an atherosclerotic plaque? (A) γ-Interferon (B) Complement (C) Interleukin-6 (D) Natural killer cells (E) Platelet-derived growth factor

*The answer is D.* In many cases of primary carnitine deficiency, increasing the blood levels of carnitine is sufficient to allow some transport of carnitine into cells such that fatty acid oxidation can occur. While pantothenic acid (part of coenzyme A), niacin (the precursor for NAD+), and riboflavin (needed for FAD) are required for fatty acid oxidation, the rate-limiting step in these patients is the transport of the fatty acids from the cellular cytoplasm to the matrix of the mitochondria.

A 6-month-old baby was doing well until he developed viral gastroenteritis and was unable to tolerate oral feeding for 2 days. He is admitted to the hospital with encephalopathy, cardiomegally and heart failure, poor muscle tone, and hypoketotic hypoglycemia. Blood work did not detect any medium-chain dicarboxylic acids. Dietary supplementation of which one of the following would be beneficial to this patient? (A) Pantothenic acid (B) Niacin (C) Riboflavin (D) Carnitine (E) Thiamine

*The answer is A.* This baby has primary carnitine deficiency, an autosomal recessive disorder. The lack of medium-chain dicarboxylic acids in the blood rules out an MCAD deficiency. He is unable to transport blood-borne carnitine into the muscle and liver, thereby blocking fatty acid oxidation in those tissues. Carnitine is required to transfer most fatty acids from the cytoplasm to the matrix of the mitochondria. However, short- and medium-chain fatty acids (up to 10 or 12 carbons) are sufficiently water-soluble such that they can enter cells and be transferred into the mitochondria in the absence of carnitine. Once inside the mitochondria, an acyl-CoA synthetase will activate the fatty acid to an acyl-CoA such that β-oxidation can occur. The transfer is not affected whether the fatty acid is saturated or unsaturated; the chain length is the determining factor. Dietary restriction of long-chain fatty acids is essential to treat this disorder and alleviate the symptoms. The patient was doing well while feeding on a regular schedule because of the carbohydrate in the diet. Once the child had an extended fast, and needed to oxidize fatty acids for energy, the symptoms of carnitine deficiency became apparent. The hypoketotic hypoglycemia is a strong indication that the problem is in fatty acid oxidation.

A 6-month-old baby was doing well until he developed viral gastroenteritis and was unable to tolerate oral feeding for 2 days. He is admitted to the hospital with encephalopathy, cardiomegally and heart failure, poor muscle tone, and hypoketotic hypoglycemia. Blood work did not detect any medium-chain dicarboxylic acids. Once this baby is diagnosed and treated, his diet will need to be very restricted. Theoretically, which one of the following fatty acids will he be able to consume and metabolize? (A) An 8-carbon fatty acid (B) A 14-carbon fatty acid (C) A 20-carbon fatty acid (D) Only unsaturated fatty acids, regardless of chain length (E) Only saturated fatty acids, regardless of chain length

*The answer is A.* The patient has a primary carnitine deficiency and can only metabolize medium-chain fatty acids. Coconut oil is high in medium-chain saturated fatty acids. Tuna and certain nuts are high in very long-chain fatty acids and omega-3 fatty acids. Spinach is a good source of ALA (alpha-linolenic acid), and omega-6 fatty acids. Oleic acid is a cis-Δ9 C18:1 fatty acid, and would not be metabolized in a child lacking carnitine in the cells.

A 6-month-old baby was doing well until he developed viral gastroenteritis and was unable to tolerate oral feeding for 2 days. He is admitted to the hospital with encephalopathy, cardiomegally and heart failure, poor muscle tone, and hypoketotic hypoglycemia. Blood work did not detect any medium-chain dicarboxylic acids. Which one of the following foods or supplements would be allowable on the above patient's restricted diet? (A) Coconut oil (B) Tuna (C) Walnuts (D) Spinach (E) Oleic acid supplements

*The answer is E.* The child has a mutation in the enzyme which transports carnitine into liver and muscle cells, leading to a primary carnitine deficiency. The carnitine stays in the blood and is eventually lost in the urine (the same carnitine transporter is required to recover the carnitine from the urine in the kidney). Since the liver is carnitine deficient, ketone body production is minimal at all times, even during a fast (thus, the lack of baseline ketone bodies in the circulation under these conditions). Fatty acids will rise in circulation, as they cannot be stored in the cells as acyl-CoA. The liver shows evidence of triglyceride formation as the acyl-CoA cannot be degraded, and acyl-CoA accumulates within the cytoplasm, leading to triglyceride formation. A defect in carnitine acyl transferase 1 would lead to elevated levels of carnitine in the circulation. A defect in carnitine acyltransferase II would lead to elevated levels of acyl-carnitine in the circulation (since the acyl group cannot be removed from the carnitine). The lack of circulating dicarboxylic acids indicates that the defect is not in MCAD (medium-chain acyl-CoA dehydrogenase). A defect in hormone sensitive lipase would show a decrease in free fatty acid levels, rather than the increase observed in the patient.

A 6-month-old child presents to the physician in a hypotonic state. The child has previously had a number of hypoglycemic episodes, at which times blood glucose levels were between 25 and 50 mg/dl. Blood work shows normal levels of ketone bodies (not elevated) during hypoglycemic episodes. Carnitine levels in the blood were, however, below normal. Free fatty acid levels were elevated in the blood, however acyl-carnitine levels were normal. Dicarboxylic acid levels were non-detectable in the blood. A liver biopsy shows elevated levels of triglyceride. A likely enzymatic defect is which of the following? (A) Carnitine acyltransferase I (B) Carnitine acyltransferase II (C) Medium chain acyl-CoA dehydrogenase (D) Hormone sensitive lipase (E) Carnitine transporter

*The answer is B.* VLDL is produced mainly from dietary carbohydrate, LDL is produced from VLDL, and chylomicrons contain primarily dietary triacylglycerol. Elevated HDL levels are desirable and are not considered to be a lipid disorder. HDL also contains low levels of triglyceride. A low-carbohydrate diet would be expected to reduce the level of circulating VLDL due to reduced fatty acid and triglyceride synthesis in the liver.

A patient with a hyperlipoproteinemia would most likely benefit from a low-carbohydrate diet if the lipoproteins that are elevated in the blood belong to which class of lipoproteins? Choose the one best answer. (A) Chylomicrons (B) VLDL (C) LDL (D) HDL (E) Chylomicrons and VLDL (F) VLDL and LDL (G) LDL and HDL

*The answer is E.* Carnitine is a methylated derivative of lysine that is used by the enzyme carnitine palmitoyltransferase (CPT./CAT}. This enzyme catalyzes the movement of a long-chain fatty acyl group from CoA to carnitine, and then the fatty acid attached to carnitine is carried across the inner mitochondrial membrane and into the mitochondrial matrix. Within the mitochondrion, fatty acyl-coA is regenerated and undergoes beta-oxidation. Carnitine is obtained to some degree from the diet and to a greater degree by endogenous synthesis. Carnitine deficiency can result either from increased demand (e.g.. ketosis) coupled with inadequate diet or from defects in the carnitine metabolic pathways. In addition to features noted in the question stem, carnitine deficiency may cause fatty liver, lipid storage myopathy, and confusion.

A 6-month-old girl is brought to the physician because of persistent weakness. Laboratory studies show hypoglycemia, hyperammonemia, and myoglobinuria. A diagnosis of defective carnitine biosynthesis is mane. This molecule is necessary for which of the following biochemical functions? A. Carrier of free fatty acids in the blood B. Importing nascent proteins into the endoplasmic reticulum C. Targeting proteins to proteasomes D. Transfer of digestive enzymes from the Golgi apparatus to lysosomes E. Transport of long-chain fatty acids into mitochondria

*The answer is B.* Niacin, or vitamin B3, is a therapy for raising HDL levels and, to a lesser extent, for lowering LDL and triglyceride levels. It has fallen out of favor recently, as these effects are relatively minimal, and its side effects can be unpleasant. Patients taking niacin describe transient cutaneous flushing episodes, which can be alleviated by taking aspirin. Severe deficiency of niacin, in contrast, results in pellagra, evidenced by dermatit is, diarrhea, glossit is, mental confusion, and eventually even dement ia.

A 60-year-old man presents to his primary care physician for a routine physical examination. Blood work reveals a decreased HDL level and he is prescribed a therapeutic substance to improve it. The physician informs the patient that to alleviate a transient common adverse effect of this therapy, he should take aspirin. Severe deficiency of the substance prescribed results in which of the following? A. Cracking of the lips and corners of the mouth and corneal vascularization B. Dermatitis, diarrhea, and a smooth, beefy-red tongue C. Dilated cardiomyopathy, edema D. Night blindness, d1y skin, and impaired immune response

*The answer is B.* After a person eats a high-fat meal, triglycerides are processed by the intestinal mucosal cells. They are assembled in chylomicrons and eventually sent into the circulation for delivery to adipocytes and other cells. Chylomicrons are too large to enter cells, but are degraded while in the circulation by lipoprotein lipase, an enzyme associated with the luminal surface of endothelial cell membranes. A defect in this enzyme would result in the accumulation of chylomicrons in the plasma, causing the plasma to appear milky (lactescence}, even in a fasting blood sample.

A 7-year-old boy is brought to the physician by his mother because of digestive problems. His mother says that he often experiences severe abdominal cramps after eating a high-fat meal. Diagnosis of a genetic defect resulting in a deficiency of lipoprotein lipase is made. Which of the following substances is most likely increased in this patient's plasma following a fatty meal? A. Albumin-bound free fatty acids B. Chylomicrons C. HDL D. LDL E. Unesterifled fatty acids

*The answer is C.* The pancreas produces bicarbonate (which neutralizes stomach acid) and digestive enzymes (including pancreatic lipase and colipase, enzymes that degrade dietary lipids). The decreased production of bicarbonate will lead to a decrease of intestinal pH. Lower levels of pancreatic lipase will result in the decreased digestion of dietary triacylglycerols, which will lead to the formation of fewer bile salt micelles. The reduced pH will also interfere with the ability of the bile salts to effectively form micelles. Intestinal cells will have less substrate for chylomicron formation, and less fat soluble vitamins will be absorbed. More dietary fat will be excreted in the feces.

A deficiency of pancreatic exocrine secretion can result in which one of the following? (A) An increased pH in the intestinal lumen (B) An increased absorption of fat-soluble vitamins (C) A decreased formation of bile salt micelles (D) Increased levels of blood chylomicrons (E) Decreased amounts of fat in the stool

*The answer is B.* Chylomicrons are produced by enterocytes (intestinal epithelial cells), using dietary gut luminal triglycerides and cholesterol as the source of the lipid. The chylomicrons are secreted into the gut lymphatic system, and from there, drain eventually into the systemic venous system from the thoracic duct, thus initially bypassing the liver and entering the peripheral blood. They are initially large and have a very high triglyceride content. With time, lipoprotein lipase releases triglycerides from the chylomicron core by hydrolyzing them to more easily absorbed fatty acids, The enzyme is located on the luminal surface of the vascular endothelium of adipose tissue for storage, and other tissues with triglyceride needs such as skeletal muscle, cardiac muscle tissue. and lactating breast. The result of lipoprotein lipase activity is that the chylomicrons shrink in size.

A healthy volunteer participates in a nutritional research study. After participants ingest a very fatty meal serum samples are taken at 1 hour and 3 hours. The average diameter of the chylomicrons is measured. showing an average chylomicron diameter of 500 nm at 1 hour, which decreases to an average diameter of 150 nm at 3 hours. At which of the following locations is the enzyme responsible for this change most likely located? A. Adipocytes B. Endothelial cells C. Enterocytes D. Hepatocytes E. Myocytes

1. *The answer is D.* A palmitate residue attached to carbon 1 of a dietary triacylglycerol is released by pancreatic lipase and carried from the intestinal lumen to the gut epithelial cell in a bile salt micelle, which will allow absorption of the fatty acid by the intestinal epithelial cell. 2. *The answer is B.* Palmitate is absorbed into the intestinal cell and utilized to synthesize a triacylglycerol, which is packaged in a nascent chylomicron and secreted via the lymph into the blood. 3. *The answer is B.* The chylomicron, containing the palmitate, matures in the blood by accepting proteins from HDL. It travels to a fat cell. VLDL is the particle made in the liver with endogenous triglyceride. 4. *The answer is C.* The chylomicron triacylglycerol is digested by lipoprotein lipase, and the palmitate enters a fat cell and is stored as triacylglycerol. It is released as free palmitate and carried, complexed with albumin, to a muscle cell, where it is oxidized.

A molecule of palmitic acid, attached to carbon 1 of the glycerol moiety of a triacylglycerol, is ingested and digested. It passes into the blood, is stored in a fat cell, and ultimately is oxidized to CO2 and H2O in a muscle cell. Choose the molecular complex in the blood in which the palmitate residue is carried from the first site to the second in each of the four questions that follow. An answer may be used once, more than once, or not at all. 1. From the lumen of the gut to the surface of the gut epithelial cell 2. From the gut epithelial cell to the blood 3. From the intestine through the blood to a fat cell 4. From a fat cell to a muscle cell (A) VLDL (B) Chylomicron (C) Fatty acid- albumin complex (D) Bile salt micelle (E) LDL

*The answer is C.* Citrate translocase is required for citrate to exit the mitochondria and enter the cytoplasm in order to deliver acetyl-CoA for fatty acid biosynthesis (see the fi gure below). Acetyl-CoA, which is produced exclusively in the mitochondria, has no direct path through the inner mitochondrial membrane. However, under conditions conducive to fatty acid biosynthesis (high energy levels, and allosteric inhibition of the TCA cycle), citrate will accumulate and leave the mitochondria. Once in the cytoplasm, citrate lyase will cleave the citrate to produce acetyl-CoA and oxaloacetate. The oxaloacetate is recycled to pyruvate, producing NADPH in the process, which is also required for fatty acid biosynthesis. A defect in either carnitine acyl transferase will not affect fatty acid biosynthesis, as those enzymes are required to transport the fatty acid into the mitochondria for its oxidation. A lack of glucose-6-phosphate dehydrogenase will not interfere with fatty acid synthesis, as malic enzyme can provide sufficient NADPH for the pathway. MCAD is involved in fatty acid oxidation and does not affect fatty acid synthesis.

A mouse model has been generated as an in vivo system for studying fatty acid synthesis. An inactivating mutation was created which led to the cessation of fatty acid synthesis and death to the mice. This mutation is most likely in which of the following proteins? (A) Carnitine acyl transferase I (B) Carnitine acyl transferase II (C) Citrate translocase (D) Glucose-6-phosphate dehydrogenase (E) Medium chain acyl-CoA dehydrogenase

*The answer is A.* At 28 weeks, this neonate's lungs have not had the opportunity to fully develop, and are deficient in dipalmitoyl phosphatidylcholine, also known as surfactant. Surfactant, synthesized by type II pneumocytes, is essential in stabilizing air expanded alveoli by decreasing surface tension, thus preventing lung collapse during expiration. Surfactant is usually made most abundantly after the 35th week of gestation. The most common way to determine lung maturity is by the lecithin:sphingomyelin ratio of the amniotic fluid. If this ratio is >2.0, then the risk of developing neonatal respiratory distress syndrome (NRDS) is significantly decreased.

A neonate born at 28 weeks' gestation is having difficulty breathing. On physical examination, the neonate's heart rate is 120/min, blood pressure is 100/60 mm Hg, and respiratory rate is 55/min. He has nasal flaring and subcostal retractions. Which of the following components is deficient in this infant? (A) Dipalmitoyl phosphatidylcholine (B) Elastase (C) Functional cilia (D) Phosphatidylglycerol (E) Sphingomyelin

*The answer is A.* Acetyl CoA carboxylase deficiency drastically alters the ability of the patient to synthesize fatty acids. The fact that the infant was born at all is due to the body's ability to utilize fatty acids provided to it. However, all processes dependent upon de novo fatty acid biosynthesis are affected. The lungs, in particular, require surfactant, a lipoprotein substance secreted by alveolar type II cells, to function properly. Surfactant lowers alveolar surface tension, facilitating gas exchange. It contains significant amounts of dipalmitoyl phosphatidylcholine. Palmitate is the major end product of de novo fatty acid synthesis. Acetyl CoA carboxylase formation of malonyl CoA is the first step of fatty acid synthesis. Biotin deficiency cannot be the problem because pyruvate carboxylase in gluconeogenesis is not affected. None of the other answers listed would result in all of the symptoms given.

A newborn infant has severe respiratory problems. Over the next few days, it is observed that the baby has severe muscle problems, demonstrates little development, and has neurological problems. A liver biopsy reveals a very low level of acetyl CoA carboxylase, but normal levels of the enzymes of glycolysis, gluconeogenesis, the citric acid cycle, and the pentose phosphate pathway. What is the most likely cause of the infant's respiratory problems? a. Low levels of phosphatidyl choline b. Biotin deficiency c. Ketoacidosis d. High levels of citrate e. Glycogen depletion

*The answer is B.* Statins inhibit HMG-CoA reductase, thereby reducing intracellular cholesterol levels. The reduced intracellular cholesterol induces the expression of the LDL receptor, which binds LDL and internalizes it within the cell, thereby reducing circulating cholesterol levels, and reducing the risk of developing atherosclerosis. The inhibition of HMG-CoA reductase reduces mevalonate production, which is the precursor for isoprene biosynthesis. Isoprenes are required for the synthesis of coenzyme Q, as well as dolichol phosphate. Thus, taking statins may reduce endogenous coenzyme Q levels. HMG-CoA reductase does not play a role in intestinal cholesterol absorption, nor will it affect the production of chylomicrons, which carry dietary (exogenous) cholesterol throughout the body.

A patient is taking a statin primarily to reduce the risk of atherosclerosis. A potential problem of statin treatment is which one of the following? (A) Reduced intracellular cholesterol (B) Reduced synthesis of coenzyme Q (C) Reduced synthesis of intracellular phospholipids (D) Reduced intestinal absorption of cholesterol (E) Reduced levels of chylomicrons

*The answer is D.* The class of drugs known as the statins (e.g., lovastatin) lower blood cholesterol levels through the induction of LDL receptor expression on the liver and peripheral tissue cell surface. Statins directly inhibt HMG-CoA reductase, a key regulatory enzyme in cholesterol biosynthesis, which reduces intracellular cholesterol levels. The reduction of intracellular cholesterol leads to the induction of LDL receptors, as the cells now need to obtain their cholesterol from the circulation. Ezetimibe inhibits the intestinal absorption of cholesterol. Statins do not inhibit lipoprotein lipase or citrate lyase.

A patient with high blood cholesterol levels was treated with lovastatin. This drug lowers blood cholesterol levels due primarily to which one of the following? (A) Inhibition of absorption of dietary cholesterol (B) Inhibition of lipoprotein lipase in adipose tissue (C) Inhibition of citrate lyase in the liver (D) Induction of LDL receptors in the liver and peripheral tissues (E) Inhibition of HMG-CoA reductase in the liver and peripheral tissues

*The answer is A.* Of the blood lipoproteins, LDL contains the highest concentration of cholesterol and lowest concentration of triacylglycerols. Elevation of blood LDL levels (the result of decreased endocytosis of LDL) would result in high blood cholesterol levels and relatively normal triacylglycerol levels. A decreased ability to degrade the triacylglycerols of chylomicrons or to convert VLDL to IDL, as well as an increased ability to produce VLDL, would all result in elevated triacylglycerol levels. An inactive lipoprotein lipase would keep chylomicron and VLDL levels elevated, so an increased triglyceride level would have been seen if this were the defect.

A person with a known hyperlipoproteinemia went in for a scheduled checkup. The lab values revealed a blood cholesterol level of 360 mg/dL (the recommended level is below 200 mg/dL) and a blood triglyceride (triacylglycerol) level of 140 mg/dL (the recommended level is below 150 mg/dL). The hyperlipoproteinemia is most likely due to which one of the following? (A) A decreased ability for receptor-mediated endocytosis of LDL (B) A decreased ability to degrade the triacylglycerols of chylomicrons (C) An increased ability to produce VLDL (D) An inactive lipoprotein lipase (E) A decreased ability to convert VLDL to IDL

*The answer is A.* Bacteria in the intestine deconjugate and dehydroxylate bile salts, converting them to secondary bile salts. Therefore, the bile salts become less water-soluble and less effective as detergents, less readily absorbed, and more likely to be excreted in the feces than recycled by the liver. Fewer micelles would be produced, so less dietary lipid (including the fat- soluble vitamins) would be absorbed. Because fewer bile salts would return to the liver, more bile salts would be synthesized. Bile salts inhibit the 7α hydroxylase that is involved in their synthesis. In addition, the person's food intake might decrease, which would augment some of the effects noted above. Charges on bile salts will not affect the uptake of water soluble vitamins. Since the bacteria in the gut deconjugate and dehydroxylate the bile salts, the amount of conjugated bile salts in the intestine will decrease.

A person with an intestinal infection caused by a proliferation of bacteria in the gut would most likely have an increase in which one of the following? (A) The synthesis of bile salts in the liver (B) The amount of conjugated bile salts in the intestine (C) The absorption of dietary lipid by intestinal cells (D) Body stores of fat-soluble vitamins (E) Body stores of water-soluble vitamins

*The answer is D.* The premature infant is experiencing respiratory distress syndrome, which is caused by a deficiency of lung surfactant. The lung cells do not begin to produce surfactant until near birth, and premature infants frequently are not producing sufficient surfactant to allow the lungs to expand and contract as needed. The surfactant is composed of a number of hydrophobic proteins and dipalmitoylphosphatidylcholine. Sphingomyelin, gangliosides, triglyceride, and prostaglandins are not components of the surfactant. The phosphatidylcholine content of the surfactant is 85% of the total lipids associated with the complex.

A premature infant, when born, had low Apgar scores and was having difficulty breathing. The NICU physician injected a small amount of a lipid mixture into the child's lungs, which greatly reduced the respiratory distress the child was experiencing. In addition to proteins, a key component of the mixture was which one of the following? (A) Sphingomyelin (B) A mixture of gangliosides (C) Triacylglycerol (D) Phosphatidylcholine (E) Prostaglandins E and F

*The answer is A.* Phosphatidylcholine is the dominant component of pulmonary surfactant. Pulmonary surfactant consists of phospholipids (85%), proteins (10%), and neutral lipids (5%). About 75% of the phospholipids in surfactant are composed of phosphatidylcholine. The role of surfactant is to keep the alveoli from collapsing. It is not produced in adequate amounts until around the 36th week of gestation; thus premature infants are at risk for neonatal respiratory distress syndrome. If delivery can be delayed, the mother can be given glucocorticoids, which can help induce production of surfactant in the fetus.

A premature newborn is brought to the clinic because he is having difficulty breathing. Physical examination reveals he is breathing rapidly and has intercostal retractions, nasal flaring, and cyanosis. Endotracheal administration of an agent is performed to treat the patient's condition. Which of the following is the dominant component of this substance? A. Phosphatidylcholine B. Phosphatidylethanolamine C. Phosphatidylglycerol D. Phosphatidylinositol E. Sphingomyelin

*The answer is B.* This child has hypoketotic hypoglycemia after a period of fasting. These findings are consistent with a defect in fatty acid β-oxidation in the mitochondria. The most common enzymatic defect leading to impaired β-oxidation is acyl-CoA dehydrogenase deficiency. Normal β-oxidation of fatty acids yields FADH and NADH for ATP production and generates acetyl-CoA for the citric acid cycle and ketone bodies. During periods of fasting, patients with acyl-CoA deficiency cannot oxidize fatty acids for energy or produce ketone bodies. This results in the characteristic hypoketotic hypoglycemic. There are several variants of acyl-CoA dehydrogenase that metabolize different length fatty acid chains (very long-chain, long-chain, medium-chain, or short chain). However, the clinical findings are similar regardless of the type of acyl-CoA dehydrogenase deficiency. Carnitine deficiency prevents fatty acids from being transported into the mitochondria for β-oxidation and causes similar features. Affected patients may remain asymptomatic for long periods until they experience a significant fast, therefore, fatty acid oxidation disorders and carnitine deficiency have been added to newborn screening.

A previously healthy 8-year-old boy is brought to the emergency department due to vomiting and lethargy. The patient had been on an overnight hiking trip with his family. During the trip, the family lost their food pack while canoeing. They had to hike back to the car. The child became weak and was carried the last mile. No one has eaten for approximately 24 hours. On examination, he appears listless. Mild hepatomegaly is noted. Laboratory results are as follows: Glucose 30 mg/dL Acetoacetate not detected Aspartate Aminotransferase 341 U/L Alanine Aminotransferase Aminotransferase A. Acetyl-CoA carboxylase B. Acyl-coA dehydrogenate C Acid alpha-glucosidase D. Glucose 6-phosphatase E. Glycogen phosphorylase

*The answer is C.* The most likely cause of the symptoms observed is carnitine deficiency. Under normal circumstances, long-chain fatty acids coming into muscle cells are activated as acyl coenzyme A and transported as acyl carnitine across the inner mitochondrial membrane into the matrix. A deficiency in carnitine, which is normally synthesized in the liver, can be genetic; but it is also observed in preterm babies with liver problems and dialysis patients. Blockage of the transport of long-chain fatty acids into mitochondria not only deprives the patient of energy production, but also disrupts the structure of the muscle cell with the accumulation of lipid droplets. Oral dietary supplementation usually can effect a cure. Deficiencies in the carnitine acyltransferase enzymes I and II can cause similar symptoms.

A teenage girl is brought to the medical center because of her complaints that she gets too tired when asked to participate in gym class. A consulting neurologist finds muscle weakness in the girl's arms and legs. When no obvious diagnosis can be made, biopsies of her muscles are taken for tests. Chemistries reveal greatly elevated amounts of triacylglycerides esterified with primarily long-chain fatty acids. Pathology reports the presence of significant numbers of lipid vacuoles in the muscle biopsy. Which one of the following is the most likely diagnosis? a. Fatty acid synthase deficiency b. Tay-Sachs disease c. Carnitine deficiency d. Biotin deficiency e. Lipoprotein lipase deficiency

*The answer is C.* During alcohol ingestion by humans, liver alcohol dehydrogenase converts ethanol to acetaldehyde. The acetaldehyde can be metabolized to acetate by acetaldehyde dehydrogenase, then to acetyl CoA. The alcohol and acetaldehyde dehydrogenases both generate NADH, increasing the NADH/NAD+ ratio and stimulating lipid synthesis. The increased lipid synthesis with chronic ethanol ingestion contributes to the fatty liver of alcoholism. In normal glycolysis, three-carbon metabolites such as glyceraldehyde are metabolized to pyruvate. In the liver and in resting muscle, almost all pyruvate produced is converted to acetyl CoA for oxidation in the citric acid cycle. In actively contracting muscle, when oxygen is limited, lactate accumulates. No acetate is converted to acetaldehyde since the acetaldehyde dehydrogenase is irreversible. This reverse pathway is utilized by yeast to produce ethanol during fermentation of grapes or other plant products. Yeast converts pyruvate to acetaldehyde, then acetaldehyde to ethanol.

After alcohol ingestion, which of the following intermediates accumulates in liver that is not typical of glycolysis or the citric acid cycle? a. Acetyl CoA b. Lactate c. Acetaldehyde d. Citrate e. Oxaloacetate

*The answer is D.* Dietary glucose is the major source of carbon for synthesizing fatty acids in humans. In a high-carbohydrate diet, excess carbohydrates are converted to fat (fatty acids and glycerol) in the liver, packaged as VLDL, and sent into the circulation for storage in the fat cells. The new diet has reduced dietary lipids, which lower chylomicron levels, but the excess carbohydrate in the diet is leading to increased VLDL synthesis and elevated triglyceride levels. Dietary amino acids are usually incorporated into proteins, particularly in a low-protein diet.

An individual has been determined to have hypertriglyceridemia, with a triglyceride level of 350 mg/dL (normal is <150 mg/dL). The patient decides to reduce this level by keeping his caloric intake the same, but switching to a low-fat, low-protein, high-carbohydrate diet. Three months later, after sticking faithfully to his diet, his triglyceride level was 375 mg/dL. This increase in lipid content is being caused by which component of his new diet? (A) Phospholipids (B) Triglycerides (C) Amino acids (D) Carbohydrates (E) Cholesterol

*The answer is E.* This patient has the clinical findings of abetalipoproteinemia resulting from a mutation in the gene that encodes microsomal triglyceride transfer protein (MTP). MTP is essential for the packaging of hepatic triglycerides with other major components of very low density lipoproteins {VLDLs). This autosomal recessive disease results in extremely low cholesterol and triglyceride levels and an absence of chylomicrons VLDL, and low-density lipoproteins (LDLs}. The clinical manifestations stem from defects in the absorption and transport of fat-soluole vitamins and essential fatty acids. Vitamin E deficiency results in hyporeflexia, distal muscle weakness. ataxia. decreased vibratory sensation and proprioception, and night blindness. Other clinical features include diarrhea fat malabsorption, failure to thrive, pigmented retinopathy. and acanthocytosis. Symptoms and signs develop in childhood. Treatment consists of low-fat. high-calorie diets that are enriched with large supplemental doses of fat-soluble vitamins

An 11-year-old boy is brought to the physician because of difficulties with balance and night vision. His mother says that he had foul-smelling stools and had failure to thrive as an infant. Physical examination shows poor muscle coordination, ataxia, and decreased proprioception and vibratory sensation. Laboratory studies show low total cholesterol and vitamin E levels. This patient most likely has an inherited mutation in which of the following? A. CYP 7A (7-alpha hydroxylase) B. Hormone-sensitive lipase (HSL) C. Lecithin-cholesterol acyltransferase (LCAT) D. Lipoprotein lipase (LPL) E. Microsomal transfer protein (MTP)

*The answer C.* The child is exhibiting the symptoms of either Tay Sachs or Sandhoff's disease, both of which are sphingolipidoses. The hex A gene codes for hexosaminidase A, whereas the hex B gene codes for hexosaminidase B. The hex A protein consists of two A and two B subunits, and cleaves only GM2. The hex B protein is a B tetramer, and cleaves both GM2 and globoside. In Tay-Sachs disease, a loss of hex A activity, globoside degradation is normal as the hex B protein is normal. The loss of hex B activity affects both hex A (since two subunits are of the B variant) and hex B (tetramer) activity, and globoside will accumulate in Sandhoff's disease, but not in Tay-Sachs disease.

An 8-month-old baby girl had normal growth and development for the first few months, but then progressively deteriorated with deafness, blindness, atrophied muscle, inability to swallow, and seizures. Early on in the diagnosis of the child, it was noticed that a cherry red macula was present in both eyes. Considering the child in the above case, measurement of which one of the following would enable one to determine whether the mutation were in the hex A or hex B gene? (A) GM1 (B) GM2 (C) Globoside (D) Glucocerebroside (E) Ceramide

*The answer is E.* Ethanol metabolism (which produces high NADH levels) does not prevent glycogen degradation. In fact, glycogen stores would be rapidly depleted under these conditions because of decreased gluconeogenesis. Lactate is converted to pyruvate during gluconeogenesis. The pyruvate-lactate equilibrium greatly favors lactate when NADH is high. Thus, alanine and lactate are prevented from producing glucose. Lactate levels are elevated, and a lactic acidosis can result. Glycerol normally enters gluconeogenesis by forming glycerol-3-phosphate, which is oxidized to DHAP. High NADH levels prevent this oxidation. Aspartic acid is converted to oxaloacetate (via transamination), as do other amino acid degradation products that enter the TCA cycle (α-ketoglutarate, succinyl-CoA, fumarate). However, the high NADH levels favor malate formation from oxaloacetate, reducing the amount of oxaloacetate available for gluconeogenesis (through the phosphoenolpyruvate carboxykinase reaction). Thus, the three major gluconeogenic precursors (alanine, glycerol, and lactate) do not form glucose because of the high NADH levels, and as glycogen stores are depleted, hypoglycemia results.

An alcoholic who went on a weekend binge without eating any food was found to have severe hypoglycemia. Hypoglycemia occurred because the metabolism of ethanol prevented the production of blood glucose from which one of the following? Choose the one best answer. (A) Glycogen (B) Lactate (C) Glycerol (D) Oxaloacetate (E) Lactate, glycerol, and oxaloacetate

*The answer is C.* The acyl-CoA dehydrogenases catalyze the first step of the fatty acid oxidation spiral in that these enzymes create a carbon-carbon double bond between carbons 2 and 3 of the fatty acyl-CoA, generating an FADH2 in the process. The FADH2 then donates its electrons to the electron transfer flavoprotein (ETF), which then transfers the electrons to coenzyme Q (via the ETF:CoQ oxidoreductase). A lack of the oxidoreductase activity will lead to an accumulation of mitochondrial FADH2, depleting FAD levels, and reducing the activity of the acyl-CoA dehydrogenases. The lack of FAD does not directly inhibit the β-ketothiolase or enoyl-CoA dehydrogenase steps, nor does it affect the activity of the carnitine acyltransferases.

An inactivating mutation in the ETF:CoQ oxidoreductase will lead to an initial inhibition of which of the following enzymes in fatty acid oxidation? (A) Carnitine acyltransferase I (B) Carnitine acyltransferase II (C) Acyl-CoA dehydrogenase (D) Enoyl-CoA dehydrogenase (E) β-keto thiolase

*The answer is E.* An inactivating mutation in acetyl-CoA carboxylase would lead to an inability to produce malonyl-CoA, which regulates fatty acid oxidation through an inhibition of carnitine acyl transferase 1. As malonyl-CoA levels increase, fatty acid oxidation is reduced, and as the levels decrease, fatty acid oxidation will increase. If malonyl-CoA decarboxylase were inactivated, malonyl-CoA levels would remain elevated, and fatty acid oxidation would be inhibited. Inactivating mutations in either carnitine acyltransferase 1 or 2 would lead to an inability to oxidize fatty acids, as they would not enter the mitochondria. A defect in medium-chain acyl-CoA dehydrogenase (MCAD) would also result in reduced fatty acid oxidation, as the initial step of the oxidation spiral would be inhibited once the fatty acid had been reduced to about 10 carbons in length.

An individual contains an inactivating mutation in a particular muscle protein, which leads to weight loss due to unregulated muscle fatty acid oxidation. Such an inactivated protein could be which of the following? (A) Malonyl-CoA decarboxylase (B) Carnitine acyl transferase I (C) Carnitine acyl transferase II (D) Medium chain acyl-CoA dehydrogenase (E) Acetyl-CoA carboxylase 2

*The answer is E.* Biotinidase is required to remove covalently-bound biotin from proteins, which is how most of the biotin in our diet is received. In the absence of biotinidase, individuals can become functionally biotin-deficient, due to the lack of free biotin in the body (as compared to being covalently bound to proteins). The formation of malonyl-CoA, via acetyl-CoA carboxylase, requires biotin as a required cofactor. Citrate lyase, malic enzyme, acetyl transacylase (an activity of fatty acid synthase) and acyl carrier protein (another component of fatty acid synthase) do not require biotin for their activity.

An individual with a biotinidase deficiency was shown to produce fatty acids at a greatly reduced rate (in the absence of supplements) as compared to someone who did not have the deficiency. This is due to which of the following? (A) Low activity of citrate lyase (B) Reduced activity of malic enzyme (C) Reduced activity of acetyl transacylase (D) Defective acyl carrier protein (E) Reduced ability to form malonyl-CoA

*The answer is B.* Leptin is a protein hormone that plays an important role in regulating appetite and metabolism. It is produced primarily in adipocytes, and large fat cells produce more leptin than small ones. Serum leptin concentrations are highly correlated with body fat content. Leptin decreases food intake in the following important ways; 1. Leptin decreases the production of neuropeptide Y, a potent appetite stimulant in the arcuate nucleus of the hypothalamus 2. Leptin stimulates the production of proopiomelanocortin (POMC) in the arcuate nucleus. Alpha-melanocyte-stimulating hormone (alpha MSH) is produced by cleavage of POMC and inhibits food intake. The knockout mouse described is homozygous for a mutation in the gone encoding the leptin receptor (db/db), resulting in ineffective leptin signaling. As a result. these mice become hyperphagic and profoundly obese. As leptin production is normal in these mice, leptin levels are elevated due to the increased lipocyte mass. In contrast, mice that are homozygous for a mutation resulting in impaired leptin production (ob/ob) also become hyperphagic and profoundly obese, but their leptin levels are low (Choice A). Human obesity resulting from mutations in the leptin receptor and the leptin gene has been described. However, most obese individuals do not have either of these mutations. Instead, it is thought that the sustained elevation In leptin levels from the enlarged fat stores results in leptin desensitization. Thus, obese individuals become resistant to the effects of leptin in a manner similar to the development of insulin resistance in type 2 diabetes.

An investigator is studying weight regulation using experimental mouse models. Knockout mice are created with a homozygous mutation in the gene coding for the leptin receptor. This mutation prevents the receptor from binding leptin end initiating its normal signaling cascade. The knockout mice are allowed to feed at will, and their body mess index (BMI) and serum leptin levels are measured and compared with control mice. On the graph below. Area C represents the normal relationship between BMI and leptin in a control mouse. Which of the following areas represents the expected relationship between BMI and serum leptin levels in a leptin receptor mutant mouse?

*The answer is A.* Arachidonic acid is a substrate for lipoxygenase and for cyclooxygenase. The cyclooxygenase pathway produces thromboxanes and prostaglandins. Lipoxygenase produces leukotrienes. Aspirin and indomethacin inhibit cyclooxygenase, thereby preventing the formation of prostaglandins and thromboxanes. Leukotriene B4 is formed from the 5-lipooxygenase pathway and is primarily involved in neutrophil chemotaxis and activation. The other leukotrienes (C4, D4, and E4) are involved in vasoconstriction, bronchospasm, and increased vascular permeability. Inhibition of the cyclooxygenase pathway shunts more arachidonic acid to the lipoxygenase pathway, thereby increasing production of leukotriene B4.

Arachidonic acid is released from cell membranes in response to stimuli and is degraded by enzymes to form end-products that influence inflammation and hemostasis. Production of which of the following arachidonic acid products is increased by taking aspirin? A. Leukotriene B4 B. Prostacyclin C. Prostaglandin D2 D. Prostaglandin E2 E. Thromboxane A2

*The answer is C.* The principal pathway for hepatic metabolism of ethanol is thought to be oxidation to acetaldehyde in the cytoplasm by alcohol dehydrogenase. Acetaldehyde is then oxidized, chiefly by acetaldehyde dehydrogenase within the mitochondrion, to yield acetate. Acetone, methanol, hydrogen peroxide, and glycerol do not appear in this biodegradation pathway. The genetic variations of acetaldehyde dehydrogenase have few phenotypic effects aside from sensitivity to alcoholic beverages, and are extremely common in the affected populations. These= characteristics qualify acetaldehyde dehydrogenase variation as an example of enzyme polymorphism.

Asians and Native Americans may flush and feel ill after drinking small amounts of ethanol in alcoholic beverages. This reaction is due to genetic variation in an enzyme that metabolizes the liver metabolite of alcohol, which is a. Methanol b. Acetone c. Acetaldehyde d. Hydrogen peroxide e. Glycerol

*The answer is B.* Thromboxanes promote platelet aggregation, and aspirin blocks this function through reducing the synthesis of thromboxanes. This decreases the chances of a clot forming in a coronary artery (MI) or in the artery that feeds the brain (CVA). Aspirin also inhibits prostaglandin synthesis, but this is an anti-inflammatory property. Leukotrienes are involved in allergies and asthma, and their synthesis requires lipoxygenase, which is not inhibited by aspirin. Arachidonic acid is derived from linoleic acid, and that synthesis (fatty acid elongation) is not inhibited by aspirin. Linolenic acid is an essential fatty acid, and cannot be synthesized by humans.

Aspirin is used in small daily doses to help prevent heart attacks and/or strokes. Aspirin can be used in this fashion because it inhibits which one of the following? (A) Prostaglandin synthesis (B) Thromboxane synthesis (C) Arachidonic acid synthesis (D) Leukotriene synthesis (E) Linolenic acid synthesis

*The answer is B.* Primary carnitine deficiency is a lack of carnitine within the cell (such as a mutation in the carnitine transporter); secondary carnitine deficiency occurs when the carnitine is sequestered in the form of acyl-carnitine (the carnitine cannot be removed from the acyl group, such as a defect in carnitine acyl transferase 2). Thus, elevated levels of acylcarnitine would be expected in a secondary carnitine deficiency, but not in a primary carnitine deficiency. In both types of carnitine deficiencies, fatty acid oxidation is significantly reduced, so the levels of ketone bodies, glucose, lactate, and fatty acids would be similar under both conditions.

Carnitine deficiency can occur in a number of ways. Secondary carnitine deficiency can be distinguished from primary carnitine deficiency by measuring which of the following in the blood? (A) Fatty acids (B) Acyl-carnitine (C) Lactic acid (D) Glucose (E) Ketone bodies

*The answer is B.* Biotin is required for the acetyl-CoA carboxylase reaction in which the substrate, acetyl-CoA, is carboxylated by the addition of CO2 to form malonyl-CoA. This reaction occurs in the cytosol. Malonyl-CoA provides the 2-carbon units that add to the growing fatty acid chain on the fatty acid synthase complex. As the growing chain is elongated, malonyl-CoA is decarboxylated. Citrate is an allosteric activator of the enzyme, and the enzyme is inhibited by phosphorylation by the AMP-activated protein kinase.

Choose the one best answer that most accurately describes some properties of acetyl-CoA carboxylase.

*The answer is B.* In humans, ethanol is cleared from the body by oxidation catalyzed by two NAD+-linked enzymes: alcohol dehydrogenase and acetaldehyde dehydrogenase. These enzymes act mainly in the liver to convert alcohol to acetaldehyde and acetate, respectively. In chronic alcoholics, alcohol dehydrogenase may be elevated somewhat. The NADH level is significantly increased in the liver during oxidation of alcohol, owing to the consumption of NAD+. This leads to a swamping of the normal means of regenerating NAD+. Thus, NAD+ becomes the rate-limiting factor in the oxidation of excess alcohol.

Chronic alcoholics require more ethanol than do nondrinkers to become intoxicated because of a higher level of specific enzyme. However, independent of specific enzyme levels, the availability of what other substance is rate-limiting in the clearance of ethanol? a. NADH b. NAD+ c. FADH d. FAD+ e. NADPH

*The answer is A.* Saturated fats do not liquefy until a much higher temperature than that at which monounsaturated or polyunsaturated fats do (the melting temperature for saturated fats is greater than that for unsaturated fats). Conversely, saturated fats are solids at a higher temperature than unsaturated fats and cannot exist in a liquid form at a lower temperature. Since the oil of a plant is its "lifeblood," at a lower temperature, a saturated oil would solidify and the plant would die. Saturated oil plants cannot survive in a temperate climate (Kansas) and need a tropical climate of warm temperatures all year round. Only polyunsaturated oil plants can survive in a temperate climate (corn, flax, wheat, and canola). Monounsaturated oils need a warmer climate, but not as warm as the tropics (olive, peanut). Knowing where a plant grows gives a large clue as to whether the oil will be saturated, monounsaturated, or polyunsaturated. The difference in oil content between plants appears to be an evolutionary process. Kansas soil is very rich and supports growth of most plants.

Coconut palm tress cannot survive growing outdoors in Kansas. Which of the following is the best explanation for this finding? (A) Coconut/palm oil is a saturated fat (B) Coconut/palm oil is a monounsaturated fat (C) Coconut/palm oil is a polyunsaturated fat (D) Kansas soil is not sandy enough to support growth (E) Kansas soil is too rocky to support growth

*The answer is F.* All nicotinamides and flavin nucleotides are activated electron carriers, but only NADPH can be used in anabolic (energy-requiring) reactions. It is derived from vitamin B3 (niacin). Its uses in the body include the building of fatty acids and steroids, generation of the respiratory burst in WBCs, and the recycling (reducing) of glutathione in RBCs. Glucose-6-phosphate dehydrogenase, which is the rate-limiting enzyme in the oxidative portion of the pentose phosphate pathway (shown in the diagram), is the enzyme that generates NADPH.

Congenital adrenal hyperplasia (CAH) is an autosomal recessive disorder that can take numerous forms depending on the gene involved. It occurs when one of the numerous enzymes required to convert cholesterol to cortisol is deficient or defective. Depending on the precise enzyme involved, patients with CAH may show underproduction or excessive production of gonadal steroids. Which of the following compounds is the primary reducing equivalent (ie, the electron carrier) used in tissues that synthesize steroids? A. FAD (flavin adenine dinucleotide) B. FADH2 (reduced form of flavin adenine dinucleotide) C. NAD+ (oxidized nicotinamide adenine dinucleotide) D. NADH (reduced nicotinamide adenine dinucleotide) E. NADP+ (oxidized form of nicotinamide adenine dinucleotide phosphate) F. NADPH (reduced form of nicotinamide adenine dinucleotide phosphate)

*The answer is C.* CH3CH20H) is converted to acetaldehyde (CH3CHO) (substance C) by alcohol dehydrogenase (ADH, enzyme 1). Acetaldehyde is converted to acetate (CH3C02) by acetaldehyde dehydrogenase (ALDH, enzyme 2). Both ADH and ALDH require the cofactor NAD+ (substances A and D), which is reduced to produce NADH (substances B and E). Disulfiram irreversibly binds to NAD+-binding sites on the second enzyme in this pathway, ALDH, resulting in the build-up of acetaldehyde (substance C). The accumulation of this toxic intermediate is thought to cause the adverse effects suffered by patients who ingest alcohol while also taking disulfiram.

Disulfiram inhibits the metabolism of alcohol. In patients taking disulfiram, ingestion of alcohol is followed within 30 minutes by flushing, tachycardia, nausea, vomiting, headache, vertigo, and anxiety. Below is a diagram depicting alcohol metabolism. Elevated levels of which substance in the diagram may lead to the adverse effects described above?

*The answer is A.* Arrow 1 indicates the inner mitochondrial membrane Carnitine acyltransferase II is located on the inner face of the inner mitochondrial membrane. It reforms fatty acyl-coA in the mitochondrial matrix (arrow 5) from aryl carnitine. thus preparing it for mitochondrial β-oxidation. The fatty acyl groups on carnitine are derived from fatty acyl-coA esters synthesized on the outer mitochondrial membrane, which, in turn, are made from free fatty acids circulating in the blood. Fatly acyl-CoA synthetases (choice B) are located on the outer mitochondrial membrane indicated by arrow 2 Glucose-6-phosphate dehydrogenase, the first enzyme in the pentose phosphate pathway (choice C) ; hexokinase. the first enzyme in the glycolytic pathway (choice D) ; and pyruvate kinase (choice E), which produces pyruvate from phosphoenolpyruvate in glycolysis, are all located in the cytosol, indicated by arrow 3. Arrow 4- indicates smooth endoplasmic reticulum, which is the site of several enzymes in lipid metabolism including hydroxymethyl glutaryl-CoA reductase (HMG-CoA reductase) in cholesterol synthesis, NADH-cytochrome b5 reductase, cytochrome b5, and a desaturase that together introduce double bonds (C=C) into fatty acids to produce mono- and polyunsaturated fatty acids Arrow 5 indicates the mitochondrial matrix. in which all but one (succinate dehydrogenase) of the enzymes of the citric acid cycle are located. The enzymes of β-oxidation, pyruvate dehydrogenase ketogenesis (liver), and ketogenesis (extrahepatic tissues) are also in the mitochondrial matrix.

During an investigational study, colloidal gold-labeled antibodies are used to characterize the distribution of macromolecules in subcellular compartments. This technique is used to localize enzymes involved in intermediary metabolism. An antibody to which of the following enzymes will most likely selectively lapel the area indicated by arrow 1 in the electron micrograph shown? A. Carnitine acyltransferase II B. Fatty aryl-CoA synthetase C. Glucose-6-Dhesehate dehvdrogenase D. Hecokinase E. Pyruvate kinase

*The answer is D.* Ethanol is converted to acetaldehyde, which is further oxidized to acetic acid and is then activated to acetyl-CoA. The acetyl-CoA enters the TCA cycle to generate energy, and two carbons are lost for each turn of the cycle as CO2. Thus, ethanol cannot provide carbons for the net synthesis of glucose. Ethanol is not converted to acetone, nor is it directly lost in the urine. Ethanol is primarily oxidized in the liver, and its carbons cannot be used for the biosynthesis of lysine, which is an essential amino acid for humans.

Ethanol ingestion is incapable of supplying carbons for gluconeogenesis. This is due to which of the following? (A) Ethanol is converted to acetone, and the carbons are lost during exhalation (B) Ethanol is lost directly in the urine (C) Ethanol cannot enter the liver, where gluconeogenesis predominantly occurs (D) Ethanol's carbons are lost as carbon dioxide before a gluconeogenic precursor can be generated (E) Ethanol is converted to lysine, which is strictly a ketogenic amino acid

*The answer is A.* Lung surfactant reduces surface tension of the fluid lining the alveoli to increase pulmonary compliance and facilitate exchange of gases dissolved in that fluid from inspired air into the airway epithelial cells and eventually by diffusion into the blood. Although all the other options represent properties of water or solutions, they have nothing to do with the properties of surfactant.

Infants born prematurely are at risk for respiratory distress syndrome. In such cases, it is common to administer surfactant, the purpose of which is to alter which of the following properties of water at the alveolar interface with air? A. Surface tension B. Evaporation C. Heat of vaporization D. Ionization E. Dielectric constant

*The answer is C.* COX-2 is induced during inflammatory conditions, while COX-1 is constitutively expressed. Thus, when an injury occurs, and an immune response is mounted at the site of injury, COX-2 is induced in those cells to produce second messengers that play a role in mediating the pain response. Specifically inhibiting the COX-2 isozyme will block the production of those second messengers, without affecting the normal function of COX-1. Inhibiting COX-1 may reduce the frequency of heart attacks, and inhibiting COX-2 will block prostaglandin production via the cylco-oxygenase. Recent data suggests that certain drugs that specifically block COX-2 have unwanted side effects, such as an increase in heart attacks.

Inhibitors specific for cyclooxygenase 2 (COX-2) were deemed more efficacious for certain conditions than inhibitors which blocked both COX-1 and COX-2 activities. This is due to which of the following? (A) Inhibiting COX-1 increased the frequency of heart attacks (B) Inhibiting COX-2 did not alter prostaglandin production (C) COX-2 is specifically induced during inflammation (D) Specifically inhibiting COX-2 reduces the rate of heart attacks (E) COX-1 is inducible and only expressed during wound repair, while COX-2 is expressed constitutively

*The answer is D.* Like steroid hormones, this drug binds to a cytosolic receptor to form a hormone receptor complex. The complex is transported to the nucleus, where it acts (as does a transcription factor) on DNA to increase the transcription of LDL cholesterol receptor messenger RNA, which is translated into LDL receptors. It is important to note that both steroids and this drug are able to cross the phospholipid bilayer because of their lipophilicity.

Scientists recently discovered a new drug for treating hypercholesterolemia. In vitro studies with a hepatocyte cell line revealed that the drug increases the number of LDL cholesterol receptors by acting in a manner similar to that of steroids. What is the mechanism by which this drug is acting on hepatocytes? A. Allosteric regulation B. Cell surface receptor antagonism C. G protein-cell receptor-mediated phosphorylation D. Hormone receptor complex format ion E. Proteolytic modification

*The answer is B.* Triglyceride stored in adipose tissue is metabolized to free fatty acids and glycerol by hormone sensitive lipase. Glycerol is then transported to the liver where it is phosphorylated to glycerol-3-phosphate by the liver specific enzyme glycerol kinase (Choice B). Subsequently, glycerol-3 phosphate is converted to dihydroxyacetone phosphate (DHAP) by glycerol-3-phosphate dehydrogenase. DHAP can be used to produce ATP through gtycotysis or glucose through gluconeogenesis. Glycerol in the liver can also be utilized for triglyceride synthesis. *Educational Objective:* Only the liver can utilize the glycerol produced by the degradation of triglycerides by hormone sensitive lipase. In the liver, glycerol is used for triacylglycerol synthesis, gluconeogenesis and as an intermediate in glycolysis. (Adipose tissue synthesizes the glycerol phosphate needed for triacylglycerol synthesis from dihydroxyacetone phosphate (DHAP).

Several tissues use triglyceride breakdown products as intermediates in energy generation and glucose synthesis. Which of the following is a liver-specific enzyme that facilitates such reactions? A. Acyl CoA synthetase B. Glycerol kinase C. Glucose-6-phosphate dehydrogenate D. Acetyl CoA carboxylase E. ATP-citrate lyase

*The answer is C.* Acetoacetate will react with succinyl- CoA to produce acetoacetyl-CoA and succinate (this costs 1 GTP, as the succinate thiokinase step is skipped). The acetoacetyl-CoA is converted to two acetyl-CoA, each of which can generate 10 ATP when completely oxidized (each acetyl-CoA generates 1 GTP, 3 NADH, and 1 FADH2). The sum, then, is 20 minus the 1 lost in the CoA transferase step, for a net yield of 19 ATP.

The net energy yield obtained (moles of ATP per mole of substrate oxidized) when acetoacetate is utilized by the nervous system as an alternative energy source is which of the following? Consider that acetoacetate must be oxidized to four molecules of carbon dioxide during the reaction sequence. (A) 17 (B) 18 (C) 19 (D) 20 (E) 21

*The answer is E.* Statins are lipid-lowering medications that can cause myopathy and increased levels of creatine kinase. Statins competitively inhibit HMG-CoA reductase, the enzyme that catalyzes the rate-limiting step in cholesterol synthesis, by obstructing part of the enzyme's active site. The subsequent decrease in intrahepatic cholesterol causes upregulation of hepatic LDL cholesterol receptors, ultimately lowering plasma LDL cholesterol levels. The Michaelis-Menten constant (Km) is the concentration of a substrate at half the maximum velocity of the reaction. Because statins bind to the same binding site as the substrates of the enzyme, statins increase HMG-CoA reductase's apparent Km. Well known adverse effects due to statin use include myalgia, myositis, and rhabdomyolysis.

The results of a fasting lipid profile in a 45-year-old man with diabetes reveal hypercholesterolemia. To reduce the patient's risk of death, his physician recommends lifestyle changes and initiates therapy with a lipid-lowering medication. The doctor warns the patient that this medication may be associated with muscle pain. How does this medication work? A. Decreases the apparent Michaelis Menten constant of hepatic low-density lipoprotein (LDL) cholesterol receptors B. Decreases the maximum velocity of 3-hydroxy-3-methylglutaryl coenzyme A (HMG-CoA) reductase C. Decreases the maximum velocity of hepatic low-density lipoprotein (LDL) cholesterol receptors D. Has no effect on the apparent Michaelis-Menten constant and decreases the maximum velocity of 3-hydroxy-3-methylglutaryl coenzyme A (HMG-CoA) reductase E. Increases the apparent Michaelis-Menten constant of 3-hydroxy-3-methylglutaryl coenzyme A (HMG-CoA) reductase

*The answer is B.* The synthesis of fatty acids from glucose occurs in the cytosol, except for the mitochondrial reactions in which pyruvate is converted to citrate (pyruvate to oxaloacetate, pyruvate to acetyl-CoA, and oxaloacetate and acetyl-CoA condense to form citrate). Biotin is required for the conversion of pyruvate to oxaloacetate (a carboxylation reaction), which combines with acetyl-CoA to form citrate. Biotin is also required by acetyl-CoA carboxylase. Citrate, not isocitrate, is a key regulatory compound for acetyl-CoA carboxylase. Pantothenic acid is covalently bound to the fatty acid synthase complex as part of a phosphopantetheinyl residue. During the reduction reactions on the synthase complex, the growing fatty acid chain is attached to this residue. NADPH, produced by the malic enzyme as well as by the pentose phosphate pathway, provides the reducing equivalents.

The synthesis of fatty acids from glucose in the liver is best described by which one of the following? (A) The pathway occurs solely in the mitochondria. (B) It requires a covalently bound derivative of pantothenic acid. (C) It requires NADPH derived solely from the pentose phosphate pathway. (D) The pathway is primarily regulated by isocitrate. (E) The pathway does not utilize a carboxylation reaction.

*The answer is D.* Fats (triacylglycerols) are the most highly concentrated and efficient stores of metabolic energy in the body. This is because they are anhydrous and reduced. On a dry-weight basis, the yield from the complete oxidation of the fatty acids produced from triacylglycerols is approximately 9 kcal/g, compared with 4 kcal/g for glycogen and proteins. However, under physiologic conditions, glycogen and proteins become highly hydrated, whereas triacylglyceride stores remain relatively free of water. Therefore, while the energy yield from fat stores remains at approximately 9 kcal/g, the actual yields from the oxidation of glycogen and proteins are diluted considerably. Under anhydrous physiologic conditions, fats yield about six times the energy of glycogen stores.

The yield from complete oxidation of glycogen is approximately 4 kcal/g. However, under physiologic conditions, glycogen is highly hydrated, such that the true physiologic yield is only approximately 1.5 kcal/g. Under similar physiologic conditions, what is the approximate yield from the oxidation of triacylglyceride stores? a. 1 kcal/g b. 2 kcal/g c. 4 kcal/g d. 9 kcal/g e. 24 kcal/g

*The answer is D.* Each of the genetic deficiencies listed may present with cardiomyopathy in childhood. LCAD affects the first step in β-oxidation of fatly acids in the mitochondria. Cardiac muscle and resting skeletal muscle use fatty acids and ketones as a primary energy source, thus in LCAD deficiency, muscle weakness and eventually cardiomyopathy develop. Fasting hypoglycemia and hypoketosis occurring together are strongly suggestive of a block in fatty acid oxidation. Only a carnitine uptake deficiency (choice C) or LCAD deficiency (choice D) are related to fatty acid oxidation. Since carnitine esters (long chain acyl carnitines) are accumulating in serum and in muscle tissue a carnitine uptake deficiency is ruled out.

Two 4-month-old fraternal twin boys have had progressive weakness, diminished deep tendon reflexes. and a frog-leg position since infancy, The parents say that both boys experienced seizures that were accompanied by hypoglycemia and hypoketonemia, One twin dies at 5 months of age. At 3 years of age, the surviving twin is diagnosed with cardiomegaly and laboratory tests show elevated carnitine esters in both muscle tissue and serum. Which of the following is the most likely diagnosis? A. α-L-iduronidase deficiency B. Acid maltase deficiency (α 1,4-glucosidase deficiency) C. Carnitine uptake deficiency D. Long-chain acyl CoA dehydrogenase (LCAD) deficiency E. Muscle glycogen phosphorylase deficiency

*The answer is A.* Of the enzymes on the list shown, only acetyl coenzyme A carboxylase directly produces a product, malonyl CoA, that is an allosteric regulator of another enzyme on the list. Malonyl CoA is the product of the first step in fatty acid synthesis, which occurs during a time of excess acetyl CoA due to satisfied energy and caloric needs of tissues. Malonyl CoA allosterically inhibits carnitine acyl transferase I, which is the major regulatory enzyme of fatty acid oxidation. Fatty acid oxidation occurs during a time of energy need. Thus, when fatty acids are being synthesized during a time of "plenty," the breakdown of fatty acids is shut off.

Which enzyme is an allosteric regulator of another enzyme on the list? a. Acetyl coenzyme A carboxylase b. Pancreatic lipase c. Carnitine acyltransferase I d. Diacylglycerol lipase e. Hormone-sensitive lipase

*The answer is C.* This infant is suffering from neonatal respiratory distress syndrome (NRDS), a lung disease most commonly associated with prematurity that is a result of inadequate pulmonary surfactant production. Type II pneumocytes produce endogenous surfactant, which is composed of a mixture of lipids (70% dipalmitoylphosphatidylcholine) and proteins. Surfactant lines the alveolus and decreases surface tension created by the water molecules that line it. This surface tension is why alveoli have a tendency to collapse without surfactant. Because this infant lacks sufficient surfactant to keep his alveoli open at low volumes, he experiences respiratory distress. NRDS can present with respiratory distress and hypoxemia in infants at birth; however, infants can also appear healthy at birth and develop symptoms that progress during the first 48 hours of life. Administra t ion of exogenous surfactant improves outcomes in infants with NRDS.

Two hours after delivery, a preterm boy begins to appear ill. He has an elevated respiratory rate, appears uncomfortable, and has a blue tinge to his skin. He is making grunting noises, and his nostrils are flaring. At birth the infant appeared healthy. His mother had appropriate prenatal care, and results of his cardiac examination are normal. Which of the following treatments seeks to correct the underlying deficit in lung physiology in this patient? A. Ampicillin-sulbactam B. Blood transfusion C. Exogenous phosphatidylcholine D. Surgical cardiac valve replacement E. Ventilator support

*The answer is D.* Decreased insulin levels cause fatty acid synthesis to decrease and glucagon levels to increase. Adipose triacylglycerols are degraded, and fatty acids are released. They are converted to ketone bodies in the liver, and a ketoacidosis can occur. Nonenzymatic decarboxylation of acetoacetate forms acetone, which causes the odor associated with diabetic ketoacidosis. Insulin is required for efficient glucose transport into muscle cells.

Type 1 diabetes mellitus is caused by a decreased ability of the β cells of the pancreas to produce insulin. A person with Type 1 diabetes mellitus who has neglected to take insulin injections will exhibit which one of the following? (A) Increased fatty acid synthesis from glucose in liver (B) Decreased conversion of fatty acids to ketone bodies (C) Increased stores of triacylglycerol in adipose tissue (D) Increased production of acetone (E) Increased glucose transport into muscle cells

*The answer is B.* High blood levels of triacylglycerides and fatty stools (steatorrhea) are symptoms often observed following a meal in patients with lipoprotein lipase deficiency (type I lipidemia). Normally, lipoprotein lipase delipidate the triacylglyceride-rich dietary blood lipoproteins known as chylomicrons by cleaving off fatty acids, thereby allowing their absorption into tissues. When absent or malfunctioning, blood chylomicron levels build up and back up, preventing absorption of fats from the lumen of the gut. This leads to steatorrhea. The pancreatic enzyme secreted into the gut to hydrolyze triacylglycerides is unique in that it only cleaves off the 1' and 3' fatty acids. Thus, 2' monoacylglycerides are produced and absorbed into intestinal epithelial cells, where they are reesterified with fatty acids back into triacylglycerides. The triacylglycerides are packaged into chylomicrons for transport through the lymphatic and vascular system.

What enzyme often malfunctions in diseases associated with the symptoms of high blood triacylglyceride levels and steatorrhea? a. Phospholipase D b. Lipoprotein lipase c. Thiokinase d. Acetyl coenzyme A carboxylase e. Pancreatic lipase

*The answer is E.* 3-hydroxy-3-methylglutaryl CoA is not an inhibitor of cholesterol synthesis. The compound 3-hydroxy-3-methylglutaryl CoA is formed by the condensation of acetoacetyl CoA and acetyl CoA in the synthetic pathways for both cholesterol and ketone bodies. However, the similar enzymes involved in each pathway are separated in space. 3-hydroxy-3-methylglutaryl CoA produced in mitochondria is cleaved to yield the ketone body acetoacetate, whereas that produced in the cytosol is reduced to form mevalonic acid, which goes on to form cholesterol. The two series of reactions are also separated in time. Synthesis of cholesterol occurs when excess acetyl CoA produced from carbohydrates is available. During fasting, synthesis of cholesterol is inhibited. In contrast, synthesis of ketone bodies is most rapid during fasting, when acetyl CoA is produced by the β-oxidation of mobilized fatty acids.

Which of the following correctly describes the intermediate 3-hydroxy-3-methylglutaryl CoA? a. It inhibits the conversion of cholesterol to sex steroids b. It is formed only in the cytoplasm c. It inhibits the first step in cholesterol synthesis d. It is formed by condensation of two molecules of acetyl CoA e. It is an intermediate in the synthesis of cholesterol

*The answer is H.* The regulatory enzyme of lipolysis is hormone-sensitive lipase. It is a triacylglyceride lipase of adipose cells regulated by hormones. The hormones that stimulate release of fatty acids into the blood are glucagon, epinephrine, and norepinephrine, all of which activate adipocyte membrane adenylate cyclase. This produces an increased level of cyclic AMP, which activates a protein lipase that, in turn, phosphorylates and activates the sensitive lipase. In contrast, insulin causes dephosphorylation and inhibition, thereby shutting down lipolysis and the release of fatty acids into the bloodstream.

Which of the following enzymes is activated by cAMP during fasting? a. Phospholipase D b. Lipoprotein lipase c. Thiokinase d. Acetyl coenzyme A carboxylase e. Pancreatic lipase f. Carnitine acyltransferase I g. Diacylglycerol lipase h. Hormone-sensitive lipase i. Thiolase

*The answer is B.* Lipolysis is directly regulated by hormones in adipocytes. Epinephrine stimulates adenylate cyclase to produce cyclic AMP, which in turn stimulates a protein kinase. The kinase activates triglyceride lipase by phosphorylating it. Lipolysis then proceeds and results in the release of free fatty acids and glycerol. A futile reesterification of free fatty acids is prevented, since adipocytes contain little glycerol kinase to phosphorylate the liberated glycerol, which must be processed in the liver. Inhibition of lipolysis occurs in the presence of insulin, which lowers cyclic AMP levels. Lipoprotein lipase is not an adipocyte enzyme.

Which of the following regulates lipolysis in adipocytes? a. Activation of fatty acid synthesis mediated by cyclic AMP b. Activation of triglyceride lipase as a result of hormone-stimulated increases in cyclic AMP levels c. Glycerol phosphorylation to prevent futile esterification of fatty acids d. Activation of cyclic AMP production by insulin e. Hormone-sensitive lipoprotein lipase

*The answer is D.* Fatty acids, cleaved from the triacylglycerols of chylomicrons and VLDL by the action of lipoprotein lipase, are taken up by adipose cells and react with coenzyme A to form fatty acyl-CoA. The lipoprotein lipase is not required to synthesize triglyceride within the adipocyte. Glucose is converted via DHAP to glycerol-3 phosphate, which reacts with fatty acyl-CoA to form phosphatidic acid. Adipose tissue lacks glycerol kinase and cannot use glycerol to directly form glycerol-3-phosphate. After inorganic phosphate is released from phosphatidic acid, the resultant diacylglycerol reacts with another fatty acyl-CoA to form a triacylglycerol, which is stored in the adipose cells. (2-Monoacylglycerol is an intermediate for triglyceride synthesis only in intestinal cells, and is not produced in the adipocyte.)

Which one of the following best describes the synthesis of triglyceride in adipose tissue?

*The answer is F.* During fasting, the hormone-sensitive lipase of adipose tissue is activated by a mechanism involving increased glucagon (and decreased insulin), cAMP, and protein kinase A. Phosphorylation of hormone-sensitive lipase activates the enzyme. Triacylglycerols are degraded, and fatty acids and glycerol are released into the blood. In the liver, glycerol is converted to glucose by gluconeogenesis and fatty acids are oxidized to produce ketone bodies. These fuels are released into the blood and supply energy to various tissues. *During fasting, the liver does not produce significant quantities of VLDL.* Fatty acid synthesis is reduced owing to the phosphorylation and inactivation of acetyl-CoA carboxylase by the AMP-activated protein kinase.

Which one of the following best represents fasting conditions?

*The answer is C.* An 18-carbon fatty acid will generate an additional acetyl-CoA, one NADH, and one FADH2 as compared to a 16-carbon fatty acid. Thus, the addition of two carbons will add 14 additional ATP to the overall energy yield (10 ATP per acetyl-CoA, 2.5 for NADH, and 1.5 for FADH2). An unsaturation at an odd carbon position will require the use of an isomerase during oxidation, and this will result in the loss of generation of 1 FADH2; an unsaturation at an even carbon position will require the use of the 2,4 dienoyl-CoA reductase, and this will result in the loss of generation of 1 NADPH. Thus, an unsaturation at an odd position results in the loss of 1.5 ATP, while an unsaturation at an even position results in the loss of 2.5 ATP. Thus, in comparing two 18-carbon fatty acids, one with an unsaturation at position 9, and the other at position 6, the fatty acid with the double bond at position 9 will yield one more ATP than the fatty acid with the unsaturation at position 6.

Which one of the following fatty acids will generate the largest amount of ATP upon complete oxidation to carbon dioxide and water? (A) C16:0 (B) cisΔ9 C16:1 (C) cisΔ9 C18:1 (D) cisΔ6 C18:1 (E) cisΔ9, Δ12 C18:2

*The answer is A.* Lipolysis of triacylglycerols yields fatty acids and glycerol. The free glycerol is transported to the liver, where it can be phosphorylated to glycerol phosphate and enter the glycolysis or the gluconeogenesis pathways at the level of dihydroxyacetone phosphate. Acetyl CoA and propionyl CoA are produced in the final round of degradation of an odd-chain fatty acid. Acetyl CoA cannot be converted to glucose, but propionyl CoA can. The three carbons of propionyl CoA enter the citric acid cycle after being converted into succinyl CoA. Succinyl CoA can then be converted to oxaloacetate and enter the glycolytic scheme. Ketone bodies, including β-hydroxybutyrate, are produced from acetyl CoA units derived from fatty acid β-oxidation. They may not be converted to glucose. Amino acids are not a product of triacylglycerol breakdown.

Which one of the following products of triacylglycerol breakdown and subsequent β oxidation may undergo gluconeogenesis? a. Propionyl CoA b. Acetyl CoA c. All ketone bodies d. Some amino acids e. β-hydroxybutyrate

*The answer is B.* Because chylomicrons contain the most triacylglycerol, they are the least dense of the blood lipoproteins. Because VLDL contains more protein than chylomicrons, it is more dense than chylomicrons, but less dense than LDL. Because LDL is produced by the degradation of the triacylglycerols of VLDL, LDL is denser than VLDL. HDL is the most dense of the blood lipoproteins. It has the most protein and the least triacylglycerol.

Which one of the following sequences places the lipoproteins in the order of most dense to least dense? (A) HDL/VLDL/chylomicrons/LDL (B) HDL/LDL/VLDL/chylomicrons (C) LDL/chylomicrons/HDL/VLDL (D) VLDL/chylomicrons/LDL/HDL (E) LDL/chylomicrons/VLDL/HDL

*The answer is D.* With a carnitine deficiency, fatty acids cannot be added to carnitine, and acyl-carnitine would not be synthesized. With a carnitine acyl-transferase 2 deficiency, the fatty acids are added to carnitine, but the acyl-carnitine cannot release the acyl group within the mitochondria. This will lead to an accumulation of acylcarnitine, which will lead to an accumulation in the circulation. The end result of either deficiency is a lack of fatty acid oxidation, such that ketone body levels would be minimal under both conditions, and blood glucose levels would also be similar in either condition. Insulin release is not affected by either deficiency, and carnitine levels, normally low, would not be significantly modified in either deficiency.

You are examining a patient who exhibits fasting hypoglycemia and need to decide between a carnitine deficiency and a carnitine acyltransferase 2 deficiency as the possible cause. You order a blood test to specifically examine the levels of which one of the following? (A) Glucose (B) Ketone bodies (C) Insulin (D) Acyl-carnitine (E) Carnitine

*The answer is B.* Montelukast is a leukotriene blocker. Leukotrienes are formed through the lipoxygenase pathway and affect bronchoconstriction and allergy pathways. The cyclooxygenase pathway produces prostaglandins and thromboxanes. The P450 pathway produces epoxides. The Cori cycle is related to gluconeogenesis (lactate transfer from the muscle to the liver), while the TCA cycle is utilized to oxidize acetyl-CoA to CO2 and H2O.

You have an asthmatic patient who is already on an inhaled steroid and albuterol, but is still having difficulty. You add montelukast to her regimen. Montelukast (Singulair) specifically blocks the product of which of the following metabolic pathways? (A) Cyclooxygenase (B) Lipoxygenase (C) P450 (D) Cori cycle (E) TCA cycle

*The answer is A.* Eicosanoids are potent regulators of cellular function. They are derived from arachidonic acid and are metabolized by three pathways: the cyclooxygenase pathway (prostaglandins and thromboxanes), lipoxygenase pathway (leukotrienes), and the cytochrome P450 pathway (epoxides). Nonsteroidal anti-infl ammatory drugs (NSAIDs) do not block arachidonic acid release from the membrane (which would block all eicosanoid synthesis); however, they do interfere with the cyclooxygenase pathway. Prostaglandins affect inflammation, thromboxanes affect formation of blood clots, and leukotrienes affect bronchoconstriction and bronchodilatation. NSAIDs block prostaglandins as one of their anti-inflammatory mechanisms. Thus, while NSAIDS will block both prostaglandin and thromboxane synthesis, it is the blockage of prostaglandin synthesis which will block the inflammatory symptoms.

You prescribe ibuprofen to help reduce your patient's inflammation. Which of the following pathways is blocked as an anti-infl ammatory mechanism of action of nonsteroidal anti-infl ammatory drugs? (A) Prostaglandin synthesis (B) Thromboxane synthesis (C) Leukotriene synthesis (D) All eicosanoid synthesis (E) Arachidonic acid release from the membrane

*The answer is B.* α-oxidation leads to the oxidation of the α-carbon of a branched chain fatty acid to generate an α-keto acid, which undergoes oxidative decarboxylation. This reorients the methyl groups on the branched chain fatty acid such that they are on the α-carbon, rather than the β-carbon. In this manner, the methyl groups do not interfere with the β-oxidation spiral (if the methyl group were on the β carbon, a carbonyl group would be unable to form on that carbon, which would block further oxidation of the fatty acid). Answer choices A, C, D, and E are eliminated as requiring α-oxidation because, after one round of normal β-oxidation, the methyl group (or butyl group) will be on the α-carbon and would not interfere with the β-oxidation spiral. A

α-oxidation would be required for the complete oxidation of which of the following fatty acids?


Ensembles d'études connexes

Bob Brooks National Land Use Exam Questions

View Set

NMNC 1110 EAQ 10: Safety and Infection Control (Mastery)

View Set

Chapter 9 : Skeletal Muscle Tissue Test

View Set

Forensic Science- Hair Follicles

View Set

Wordly Wise 3000® Level 7, Lesson 12

View Set

BUS 100 CH.6 the nature of management

View Set